Site Loader

Содержание

Проекция силы на ось

Решение задач на равновесие сходящихся сил с помощью построения замкнутых силовых многоугольников в большинстве случаев сопряжено с громоздкими построениями. Более общим и универсальным методом решения таких задач является переход к определению проекций заданных сил на координатные оси и оперирование с этими проекциями. Осью называют прямую линию, которой приписано определенное направление. Проекция вектора на ось является скалярной величиной, которая определяется отрезком оси, отсекаемым перпендикулярами, опущенными на нее из начала и конца вектора.  Проекция вектора считается положительной (+), если направление от начала проекции к ее концу совпадает с положительным направлением оси.  Проекция вектора считается отрицательной (—), если направление от начала проекции к ее концу противоположно положительному направлению оси.Рассмотрим ряд случаев проецирования сил на ось: Вектор силы F (рис.12,a) составляет с положительным направлением оси х острый угол а. Чтобы найти проекцию, из начала и конца вектора силы опускаем перпендикуляры на ось х; получаем: . Проекция вектора в данном случае положительна. Сила F (рис.12,б) составляет с положительным направлением оси х тупой угол а. Тогда, т. е. Fx = — F*cos р. Проекция силы F на ось х в данном случае отрицательна Сила F (рис.12,в) перпендикулярна оси х. Проекция силы F на ось х равна нулю, т.е. Fx — F cos 90° = 0.Итак, проекция силы на ось координат равна произведению модуля силы на косинус угла между вектором силы и положительным направлением оси.Силу, расположенную на плоскости хОу (рис.13), можно спроектировать на две координатные оси Ох и Оу. На рисунке изображена сила F и ее проекции Fx, Fу. Ввиду того, что проекции образуют между собой прямой угол, из прямоугольного треугольника АСВ следует: . Этими формулами можно пользоваться для определения модуля и направления силы, когда известны ее проекции на координатные оси. 

Аналитический способ нахождения равнодействующей Геометрический способ нахождения равнодействующей системы сил сопря­жен с определенными трудностями, особенно в случае большого числа сил. Поэтому предпочтительнее аналитический метод нахождения равнодействующей. Пусть система сходящихся сил на плоскости имеет равно­действующую . Обозначим через проекции этой равнодействующей на оси системы координат XOY, а через проекции сил на те же оси. Из математики известно, что проекция суммы век­торов на какую – либо ось равна алгебраической сумме проекций слагаемых векторов на ту же ось.

Пара сил

 — система двух сил F1 и F2, действующих на твёрдое тело, равных друг другу поабсолютной величине, параллельных и направленных противоположно друг другу. Пара сил не имеет равнодействующей, то есть её действие на тело не может быть механически эквивалентнодействию какой-нибудь одной силы; соответственно пару сил нельзя уравновесить одной силой.Расстояние r1+r2 между линиями действия сил пары называется плечом пары сил. Действие, оказываемое парой сил на твёрдое тело, характеризуется её моментом, который изображается вектором 

T, равным по абсолютной величине  и направленным перпендикулярно кплоскости действия пары сил в ту сторону, откуда поворот, совершаемый парой сил, виден происходящим против хода часовой стрелки (в правой системе координат). Основное свойство пары сил: действие, оказываемое ею на данное твёрдое тело, не изменяется, если пару сил переносить куда угодно в плоскости пары или в плоскости, ей параллельной, а также если изменить абсолютную величину сил пары и длину её плеча, сохраняя неизменным момент пары сил. Таким образом, момент пары сил можно считать приложенным к любой точке тела. Две пары сил с одинаковыми моментами 
T
, приложенные к одному и тому же твёрдому телу, механически эквивалентны одна другой. Любая система пар сил, приложенных к данному твёрдому телу, механически эквивалентна одной паре сил с моментом, равным геометрической сумме векторов-моментов этих пар сил. Если геометрическая сумма векторов-моментов некоторой системы пар сил равна нулю, то эта система пар сил является уравновешенной.

сЛОЖЕНИЕ ПАР СИЛ. УСЛОВИЕ РАВНОВЕСИЯ ПАР.

Теорема. Система пар, действующих на тело в одной плоскости, эквивалентна паре сил с моментом, равным алгебраической сумме моментов пар системы. Допустим, на тело действуют три пары (рис. 

а), моменты которых  известны. Каждую из заданных пар заменим эквивалентной парой соответственно но с одинаковыми плечами  т. e.  и расположим эти пары так, чтобы их силы действовали вдоль двух параллельных прямых (рис. б).Как известно, равнодействующая сил, действующих вдоль одной прямой, направлена по той же прямой и модуль ее равен алгебраической сумме составляющих сил. Поэтому, сложив силы, приложенные к точкам и к точкам , , , получим равнодействующую пару эквивалентную трем заданным парам (рис. в). При этом .Момент равнодействующей парыа так как , то  или . Теорема доказана.Распространяя равенство  на любое число пар, действующих на тело, можем записать .Следовательно, для того чтобы сложить любое число пар, действующих на тело в одной плоскости, достаточно алгебраически сложить моменты этих пар. Полученный в результате сложения момент и определяет равнодействующую пару сил.Если в результате сложения пар , то действующие на тело пары образуют уравновешенную систему. Следовательно, необходимое и достаточное условие равновесия системы пар выражается одним уравнением ,т. е. для равновесия системы пар сил, действующих на тело в одной плоскости, необходимо и достаточно, чтобы алгебраическая сумма и

Момент силы относительно точки и оси

Момент силы относительно точки О — это вектор, модуль которого равен произведению модуля силы на плечо — кратчайшее расстояние от точки О до линии действия силы. Направление вектора момента силы перпендикулярно плоскости, проходящей через точку и линию действия силы, так, что глядя по направлению вектора момента, вращение, совершаемое силой вокруг точки О, происходит по часовой стрелке.Если известен радиус-вектор 

r⃗  точки приложения силы F⃗  относительно точки О, то момент этой силы относительно О выражается следующим образом:Действительно, модуль этого векторного произведения:В соответствии с рисунком |r⃗ |sinα=h, поэтому:Вектор M⃗ O, как и результат векторного произведения, перпендикулярен векторам r⃗  и F⃗ , которые принадлежат плоскости Π. Направление вектора 
M
⃗ O таково, что глядя по направлению этого вектора, кратчайшее вращение от r⃗  к F⃗  происходит по часовой стрелке. Другими словами, вектор M⃗ O достраивает систему векторов (r⃗ ,F⃗ ) до правой тройки.Зная координаты точки приложения силы в системе координат, начало которой совпадает с точкой О, и проекцию силы на эти оси координат, момент силы может быть определен следующим образом:Момент силы относительно осиПроекция момента силы относительно точки на некоторую ось, проходящую через эту точку называется моментов силы относительно оси.Момент силы относительно оси вычисляется как момент проекции силы F⃗  на плоскость Π, перпендикулярную оси, относительно точки пересечения оси с плоскостью Π:Знак момента определяется направлением вращения, которое стремится придать телу сила 
F
⃗ Π. Если, глядя по направлению оси Oz сила вращает тело по часовой стрелке, то момент берется со знаком «плюс», иначе — «минус».

Рекомендуется такая последовательность изучения материала

ТЕХНИЧЕСКАЯ МЕХАНИКА

Методические указания и контрольные задания

для специальности13.02.11 Техническая эксплуатация и обслуживание электрического и электромеханического оборудования (по отраслям)

заочной формы обучения

 

 

Мончегорск,

 2016

Составитель– Евстрикова Татьяна Анатольевна, преподаватель ГАПОУ МО «МонПК»

 

 

Краткая аннотация

Настоящие методические указания и контрольные задания по дисциплине«Техническая механика»    составлены для обучающимся заочного отделения по специальности 13.02.11   Техническая эксплуатация и обслуживание электрического и электромеханического оборудования (по отраслям)в соответствии с программой на 96 часов. Пособие содержит программу дисциплины, перечень учебной литературы, вопросы для самопроверки, заданияя на контрольную работу и краткие методические указания по изучению тем курса и выполнению контрольной работы.

 

 

Рассмотрены на заседании цикловой комиссии _______________________

                                                                     

полное наименование цикловой комиссии

Протокол № ___ от __________

 

Председатель ЦК Н.М Шурлина.

 

 

Рецензент – И.О. Фамилия, степень (при наличии) должность, место работы (рецензия с подписью рецензента прикладывается)

 

Согласовано

Заместитель директора по УМР                   О.В. Першина

должность

 

Согласовано

Методист                                                         

 

Содержание

Пояснительная записка………………………………………………… 4
1.  Методические указания к оформлению контрольной работы…… 7
2. Методические указания по изучению учебного материала по темам и вопросы для самоконтроля ………………………………………… 8
3. Список рекомендуемой литературы ……………………………………….. 24
4. Методические указания к выполнению контрольной работы………. 24
   
Список использованной литературы…………………………………. 62

 

ПОЯСНИТЕЛЬНАЯ ЗАПИСКА

 

Учебной дисциплиной «Техническая механика» предусматривается изучение общих законов движения и равновесия материальных тел, основ расчета элементов конструкций на прочность, жесткость и устойчивость, а также деталей машин и механизмов. Дисциплина состоит из разделов: «Теоретическая механика» и «Сопротивление материалов».

Методические указания определяют общий объем знаний по дисциплине «Техническая механика», подлежащих обязательному усвоению студентами. По данной дисциплине предусматривается выполнение одной домашней контрольной работы, охватывающей все разделы учебной программы. Контрольная работа включает в себя пять задач.

Изучать дисциплину рекомендуется последовательно по темам, в соответствии с примерным тематическим планом и методическими указаниями к ним. Степень усвоения материала проверяется умением ответить на вопросы для самоконтроля, приведенные в конце темы.

Материал, выносимый на аудиторные занятия, а также перечень выполняемых практических занятий определяются учебным заведением.

На аудиторных занятиях студентов знакомят с программой дисциплины, методикой работы над материалом и выполнения домашней контрольной работы.

Варианты контрольной работы составлены применительно к действующей рабочей программе по дисциплине. Выполнение домашней контрольной работы определяет степень усвоения студентами изучаемого материала и умение применять полученные знания при решении практических задач.

Обзорные лекции проводятся по сложным для самостоятельного изучения темам программы. Проведение практических занятий предусматривает своей целью закрепление теоретических знаний и приобретение практических умений по программе учебной дисциплины.

Учебный материал рекомендуется изучать в той последовательности, которая дана в методических указаниях:

— ознакомление с методическими указаниями по темам;

— изучение программного материала по рекомендуемой литературе;

— составление ответов на вопросы самоконтроля, приведенные после каждой темы;

— решение задач из контрольной работы по каждой теме, после её изучения.

В результате изучения дисциплины студент должен:

знать:

· основные типы связей и их реакции;

· принципы освобождения тела от связей;

· условия равновесия системы сил;

· методику расчета на сжатие, срез и смятие;

· методику расчетов на прочность, жесткость и устойчивость при различных видах деформации;

· условия прочности и жесткости при различных деформациях;

· виды передач; их устройство, назначение, преимущества и недостатки, условные обозначения на схемах;

· назначение и классификацию подшипников;

· характер соединения основных сборочных единиц и деталей;

· основные типы смазочных устройств;

· типы, назначение, устройство редукторов;

уметь:

· определять направление реакции связей основных типов;

· определять реакции связей аналитическим способом;

· находить центр тяжести плоских сечений, составленных из простых геометрических фигур и профилей стандартного проката;

· определять виды нагружения и внутренние силовые факторы в поперечных сечениях;

· определять напряжения в конструкционных элементах;

· проводить расчеты на прочность при различных видах деформации;

· производить расчеты элементов конструкций на прочность, жесткость и устойчивость;

· производить расчеты на сжатие, срез и смятие;

· производить кинематические и силовые расчеты многоступенчатого привода

При изложении материала необходимо соблюдать единство терминологии, обозначений, единиц измерения в соответствии с действующими ГОСТами.

МЕТОДИЧЕСКИЕ УКАЗАНИЯ К ОФОРМЛЕНИЮ КОНТРОЛЬНОЙ РАБОТЫ

При выполнении работы следует следующие требования к оформлению контрольной работы:

1. Контрольная работа должна быть выполнена в отдельной тетради.

2. На обложке тетради указывается учебный шифр, наименование дисциплины, курс, отделение, индекс учебной группы, фамилия, имя и отчество исполнителя.

3. Работу необходимо выполнять чернилами, четко и аккуратно, либо на компьютере. Для пометок и замечаний преподавателя на каждой странице тетради оставляют поля 3 – 4 см.

4. Каждую задачу начинать с новой страницы.

5. В конце тетради оставить несколько свободных страниц для рецензии.

6. На последней странице тетради выполненной контрольной работы следует написать полностью наименование и год издания методического пособия, из которого взято задание, и используемую литературу.

7. Необходимо полностью переписать условие, составить эскиз с обозначением сил, моментов и других величин, предусмотренных условием задачи или вытекающих из решения.

8. Решение следует выполнять в общем виде, после чего проставить числовые значения в том порядке, в каком они стоят, и получить искомый результат, придерживаясь стандартных обозначений.

9. Каждое решение задачи должно быть выполнено в определенной последовательности, обосновано теоретически, пояснено необходимым текстом и краткими формулировками произведенных действий: эти действия следует располагать в таком порядке, чтобы был виден логический ход решения задачи.

10. Все вычисления в контрольных задачах следует производить в единицах СИ, тщательно проверить правильность подстановки значений, соблюдение размерности, правдоподобность полученных результатов. Если возможно, проверить правильность ответа, решив задачу вторично каким-либо иным путем.

Выполненную контрольную работу следует своевременно выслать в колледж.

После получения зачтенной контрольной работы необходимо внимательно изучить рецензию и все замечания преподавателя, обратив внимание на ошибки, доработать материал. Незачтенная работа или выполняется заново, или переделывается частично по указанию преподавателя. Зачтенные контрольные работы предъявляются на зачете. Задания, выполненные не по своему варианту, не засчитываются и возвращаются студенту.

В процессе изучения дисциплины каждый студент в зависимости от определенного для него варианта выполняет контрольную работу.

Вариант контрольного задания  соотвествует порядковому номеру в учебном журнале группы.

МЕТОДИЧЕСКИЕ УКАЗАНИЯ ПО ИЗУЧЕНИЮ УЧЕБНОГО МАТЕРИАЛА ПО ТЕМАМ И ВОПРОСЫ ДЛЯ САМОКОНТРОЛЯ

К теме 1.1.

1. Что такое материальная точка? Абсолютно твердое тело?

2. Что называется силой и каковы ее единицы?

3. Что называется системой сил? Какие системы называются эквивалентными?

4. Что называется равнодействующей и что уравновешивающей силой?

5. Как перенести силу по линии ее действия?

6. Могут ли уравновешиваться силы действия и противодействия двух тел?

7. Как формулируются аксиомы статики и следствия из них?

8. Как определяются реакции связей?

9. Какие разновидности связей рассматриваются в статике?

10. Сформулируйте правила определения направления реакций связей.

К теме 1.2.

1. Как определяется равнодействующая системы сходящихся сил, построение силового многоугольника?

2. Какая система сил называется сходящейся?    

3. Что называется проекцией силы на ось?

4. Как определить значение и знак проекции силы на оси координат?

5. В каком случае проекция силы на ось равна нулю?

6. Сколько и какие уравнения можно составить для уравновешенной плоской системы сходящихся сил?

7. В каком случае проекция силы на ось равна модулю силы?

8. Как формулируется теорема о равновесии трех непараллельных сил, лежащих в одной плоскости?

К теме 1.3.

1. Что такое пара сил? Имеет ли она равнодействующую?

2. Что такое момент пары сил?

3. Можно ли уравновесить пару сил одной силой?

4. Какие пары называются эквивалентными?

5. Каким образом производится сложение пар сил на плоскости?

6. Как формулируется условие равновесия системы пары сил?

К теме 1.4.

1. Что называется моментом силы относительно точки?

2. Как определяется знак момента силы относительно точки?

3. Что называется плечом силы? 

4. В каком случае момент силы относительно точки равен нулю?

5. Что такое главный вектор и главный момент плоской системы сил?

6. В каком случае главный вектор плоской системы сил является ее равнодействующей?

7. Как аналитически найти главный вектор и главный момент плоской системы сил?

8. В чем сходство и в чем различие между главным вектором и равнодействующей?

9. Какие уравнения можно составить для уравновешенной произвольной плоской системы сил?

10. Какие виды опор балок Вы знаете?

11. Как рационально выбрать направления осей координат и центр моментов?

К теме 1.5.

1. Что такое центр тяжести тела?

2. Как найти координаты центра тяжести треугольника и круга? Плоского составного сечения?

3. Что называется статическим моментом площади плоской фигуры и в чем он измеряется?

К теме 1.6.

1. Что изучает кинематика?

2. Дайте определение основных понятий кинематики: траектория, расстояние, путь и время.

3. Как формулируется закон движения точки и какими способами его можно задать?

4. Что называется скоростью равномерного движения точки? Что она характеризует?

5. Как направлен вектор скорости точки при криволинейном движении?

6. Как определить нормальное и касательное ускорения точки?

7. Что можно сказать о траекториях, скоростях и ускорениях точек тела, совершающего поступательное движение?

8. Дайте определение вращательного движения тела вокруг неподвижной оси. Что называется угловым перемещением тела?

9. Что называется угловой скоростью?

10. Какая связь между частотой вращения тела и угловой скоростью вращения?

11. Какое вращательное движение называется равномерным, а какое – равнопеременным?

12. Каковы зависимости между угловыми величинами (j,w,e), характеризующими вращательное движение тела, и линейными величинами (s, v, an, ar, a), характеризующими движение какой-либо точки этого тела?

13. Дайте определение силы инерции. Как она направлена? К чему приложена?

14. Как определяется работа постоянной силы на прямолинейном пути?

15. Что называется мощностью и каковы ее единицы?

16. Если на тело действуют несколько сил, то каким образом можно найти их общую работу?

17. Что называется вращающим моментом? 18. Как выражается зависимость между вращающим моментом и угловой скоростью при заданной мощности?

К теме 2.1.

1. Каковы основные задачи раздела «Сопротивление материалов»?

2. Что такое деформация?

3. Какие деформации называют упругими и какие – пластичными?

4. Какие деформации не допустимы при нормальной работе конструкций?

5. Что называется прочностью, жесткостью и устойчивостью детали или конструкции?

6. В чем сущность расчетов на прочность и жесткость?

7. В чем сущность метода сечений?

8. Можно ли установить закон распределения внутренних сил по проведенному сечению методами статики?

9. Сколько внутренних факторов может возникнуть в поперечном сечении бруса?

10. Что называется напряжением в данной точке сечения? Каковы единицы напряжения?

11. Какая существует зависимость между напряжениями r,sиt?

К теме 2.2.

1. В каком случае прямые брусья называют стержнями?

2. Как нагрузить прямой стержень, чтобы он испытывал только растяжение?

3. Что называется эпюрой продольных сил бруса?

4. Как строится эпюра продольных сил?

5. Как определить нормальное напряжение в поперечном сечении бруса?

6. Что называется эпюрой нормальных напряжений?

7. Какие поперечные сечения бруса называют опасными?

8. Что такое модуль продольной упругости и какова его размерность?

9. Какая величина в формуле Гука характеризует жесткость материала?

10. Зависит ли нормальное напряжение от материала бруса и формы поперечного сечения?

11. Зависит ли удлинение бруса от его материала?

12. Какова цель механических испытаний материалов?

13. Какой вид имеет диаграмма растяжения образца из низкоуглеродистой стали и серого чугуна?

14. Что называется пределами пропорциональности, текучести и прочности на условной диаграмме растяжения образца из низкоуглеродистой стали?

15. До какого предельного напряжения, являющегося механической характеристикой пластичного материала, можно нагружать образец, не опасаясь появления пластической деформации?

16. В каком случае в пластичных материалах можно получить наклеп? 

17. Что такое фактический коэффициент запаса прочности?

18. Какие факторы влияют на выбор требуемого коэффициента прочности?

19. Что такое допускаемое напряжение?

20. Какие расчеты можно выполнить из условия прочности?

К теме 2.3.

1. Какова зависимость между допускаемыми напряжениями растяжения, среза и смятия?

2. По каким формулам производят расчет на срез и смятие?

3. По какому сечению (продольному или поперечному) проверяют на срез призматические шпонки?

4. На каких допущениях основаны расчеты на смятие?

5. Как определяется площадь смятия, если поверхность смятия цилиндрическая, плоская?

6. Что такое чистый сдвиг?

7. Какой величиной характеризуется деформация сдвига?

8. Какая зависимость существует между передаваемой валом мощностью, вращающим моментом и угловой скоростью?

9. Как определяется крутящий момент в продольном сечении?

10. Каков закон распределения касательных напряжений по площади поперечного сечения при кручении?

11. Какая разница между крутящим и вращающим моментами? 

12. Что является геометрическими характеристиками сечения вала при кручении?

13. Какая существует зависимость между величинами E , и Gхарактеризующими упругие свойства материалов?

14. По какой формуле определяется деформация при кручении?

15. Что такое полярный момент инерции сечения бруса? По какой формуле определяется полярный момент инерции круга?

16. Что такое полярный момент сопротивления? Как он определяется для кольца?

17. Каковы геометрические характеристики сечений при деформации среза, кручения и изгиба?

18. Что такое статический момент сечения?

19. Чему равен статический момент сечения относительно центральной оси?

20. Что такое центробежный момент инерции?

21. Каковы единицы осевого момента инерции?

22. Какова связь между моментами инерции относительно параллельных осей, из которых одна является центральной?

23. Какова зависимость между осевыми и полярными моментами инерции данного сечения?

24. Какие оси, проведенные в плоскости сечения, называют главными?

25. Как определяют осевые моменты инерции сложных сечений?

К теме 2.4.

1. Какие внутренние силовые факторы возникают в поперечных сечениях бруса при его прямом поперечном изгибе?

2. Как следует нагрузить брус, чтобы получить: а) чистый прямой изгиб; б) поперечный прямой изгиб?

3. Что называется поперечной силой в поперечном сечении бруса и чему она численно равна?

4. Что такое эпюра поперечных сил и как она строится?

5. Что называется изгибающим моментом в поперечном сечении бруса и чему он численно равен?

6. Сформулируйте правило знаков для поперечных сил и изгибающих моментов.

7. Какими дифференциальными зависимостями связаны между собой изгибающий момент, поперечная сила, интенсивность равномерно распределенной нагрузки?   

8. На каких допущениях основаны выводы расчетных формул при изгибе?

9. Каков характер деформаций, возникающих при изгибе?

10. В чем сущность гипотез и допущений при изгибе?

11. Как меняются нормальные напряжения при изгибе по высоте сечения бруса?

12. Что такое жесткость сечения при изгибе?

13. Как определить напряжения в поперечном сечении при прямом изгибе?

14. Что такое осевой момент сопротивления и каковы его единицы?    

15. Какие виды расчетов можно производить из условия прочности при изгибе?

16. Какие формы поперечных сечений рациональны для балок из пластичных материалов?

Раздел 3. Детали машин

Тема 3.1. Основные положения. Цели и задачи раздела «Детали машин». Основные определения. Механизм и машина. Классификация машин. Детали машин и их классификация.Роль стандартизации в повышении качества продукции и развитии научно-технического программа.Требования, предъявляемые к машинам и их деталям. Основные критерии работоспособности и расчета деталей машин: прочность и жесткость.

 Общие сведения о передачах. Вращательное движение и его роль в машинах и механизмах. Назначение передач в машинах. Принцип работы и классификация передач. Основные кинематические и силовые соотношения для механических передач.

Фрикционные передачи. Фрикционные передачи, их назначение и классификация. Достоинства, недостатки и область применения фрикционных передач. Цилиндрическая передача гладкими катками. Основные геометрические и кинематические соотношения. Силы в передаче. Основные сведения о расчете передачи на контактную прочность и износостойкость.

Зубчатые передачи. Общие сведения о зубчатых передачах: достоинства и недостатки, область применения. Классификация зубчатых передач. Основная теорема зацепления (без вывода). Зацепление двух эвольвентных зубчатых колес: основные элементы и характеристики зацепления; взаимодействие зубьев. Стандартные параметры зубчатого зацепления без смещения. Материалы зубчатых колес. Виды разрушения зубьев.

Прямозубые цилиндрические передачи. Основные геометрические соотношения. Силы, действующие в зацеплении. Основы расчета зубьев на контактную усталость и усталость при изгибе; исходное положение расчета, расчетная нагрузка, формулы проверочного и проектного расчетов. Краткие сведения о выборе основных параметров, расчетных коэффициентов и допускаемых напряжений. Косозубые и шевронные цилиндрические передачи. Основные геометрические соотношения. Силы, действующие в зацеплении. Особенности расчета непрямозубых передач.

Основные параметры и расчетные коэффициенты. Конические передачи. Основные геометрические соотношения и силы в зацеплении.

Червячные передачи. Общие сведения о червячных передачах: достоинства и недостатки, область применения. Материалы червяков и червячных колес. Геометрические соотношения в червячной передаче. Передаточное число. Силы, действующие в зацеплении. КПД червячной передачи. Основы расчета зубьев на контактную усталость и усталость при изгибе. Формулы проверочного и проектного расчетов. Расчет вала червяка на жесткость. Краткие сведения о выборе основных параметров, расчетных коэффициентов и допускаемых напряжений. Тепловой расчет червячного редуктора.

Ременные передачи. Основные сведения о ременных передачах: устройство, достоинства и недостатки, область применения. Классификация ременных передач: типы приводных ремней и их материалы, способы натяжения ремня, основные геометрические и кинематические соотношения. Силы и напряжения в ремне, скольжение ремня на шкивах. Критерий работоспособности и понятие о расчете ременной передачи.

Цепные передачи. Общие сведения о цепных передачах: устройство, достоинства и недостатки, область применения. Приводные цепи и звездочки. Краткие сведения о подборе цепей и их проверочном расчете.

Соединения деталей машин

Неразъемные соединения. Сварные соединения: достоинства, недостатки, область применения. Основные типы сварных швов. Краткие сведения о расчете сварных соединений при осевом нагружении. Клеевые соединения: достоинства и недостатки, область применения.

Резьбовые соединения. Винтовая линия, винтовая поверхность и их образование. Основные типы резьб, их стандартизация, сравнительная характеристика и область применения, конструктивные формы резьбовых соединений. Стандартные крепежные изделия. Основы расчета резьбовых соединений при постоянной нагрузке. Материалы и допускаемые напряжения.

Валы, оси, шпоночные и зубчатые соединения. Валы и оси, их назначение, конструкции и материалы. Оси вращающиеся и неподвижные. Основы расчета валов и осей на прочность и жесткость. Типы шпоночных соединений и их сравнительная характеристика. Типы стандартных шпонок. Подбор шпонок и проверочный расчет соединения. Зубчатые (шлицевые) соединения, область применения.

Подшипники. Подшипники и подпятники скольжения: назначение, типы, область применения. Материалы деталей подшипников. Условные расчеты подшипников скольжения. Классификация подшипников качения и обзор основных типов по ГОСТу. Подбор подшипников качения по динамической грузоподъемности.

Муфты. Муфты, их назначение и краткая классификация. Краткие сведения о подборе муфт.

Вопросы для самоподготовки

Кразделу 3

1. Каковы задачи раздела «Детали машин»?

2.  Что называется машиной?  

3.  Какие признаки характеризуют машину?

4.  Какая разница между машиной и механизмом?

5.  Что следует понимать под деталью и сборочной единицей?

6. Какие требования предъявляются к машинам?

7. Что следует понимать под надежностью машин и их деталей и каковы их основные критерии работоспособности

8.  Как классифицируют механические передачи по принципу действия?    

9.  Каково назначение механических передач?

10. Почему вращательное движение наиболее распространено в механизмах и машинах?

11. Для чего применяют промежуточную передачу между двигателем и рабочей машиной?

12. По каким формулам определяют кинематические и силовые соотношения в передачах?

13. Как определяется передаточное отношение?

14. Какие виды фрикционных передач вы знаете? Каковы их достоинства и недостатки.

15. Какие устройства называют вариаторами? Каковы их достоинства и недостатки?

16. Каковы достоинства и недостатки зубчатых передач?

17. По каким признакам классифицируют эти передачи?

18. В каких случаях применяют открытые зубчатые передачи?

19. Какие передачи называют закрытыми? 

20. Какие вы знаете основные параметры зубчатой пары?

21. Каковы достоинства и недостатки косозубой передачи по сравнению с прямозубой?

22. Каково назначение конических зубчатых передач?

23. Каковы недостатки конической зубчатой передачи по сравнению с цилиндрическими?

24. Какими достоинствами и недостатками обладают червячные передачи по сравнению с зубчатыми?

25. В каких случаях применяют червячные передачи?

26. Из каких материалов изготавливают червяк и червячное колесо?

27. Как выбирают число заходов червяка?

28. Зависит ли КПД червячной передачи от числа витков червяка?

29. Какая передача называется ременной?

30. Какие применяют типы ремней?

31. Какими достоинствами и недостатками обладают ременные передачи по сравнению с другими видами передач.    

32. Каковы достоинства и недостатки цепных передач?

33. Какие различают виды приводных цепей?

34. Что является основным критерием работоспособности цепных передач?

35. Для чего применяют натяжные устройства в цепных передачах?

36. Какие соединения называются резьбовыми?

37. Как классифицируются резьбы по геометрической форме и по назначению?

38. Каковы достоинства болтового соединения?

39. В каких случаях применяют шпильки.

40. Какие материалы применяют для изготовления резьбовых деталей.

41. Какая разница между валом и осью?

42. Что называется шипом, шейкой и пятой?

43. Какие материалы применяют для изготовления валов и осей

44. Что называется подшипником?

45. Какие различают типы подшипников скольжения?

46. Какими достоинствами и недостатками обладают подшипники скольжения?

47. Из каких деталей состоят подшипники качения?

48. Для чего применяется сепаратор?

49. Каковы достоинства и недостатки подшипников качения по сравнению с подшипниками скольжения?

50. Как различают группы муфт по принципу действия и характеру работы

КОНТРОЛЬНОЙ РАБОТЫ

В задачах статики приходится рассматривать равновесие несвободных тел, лишенных возможности перемещаться в направлении действия приложенных к ним так называемых активных сил. Тела, ограничивающие движение рассматриваемого тела, называются связями. Между телом и связью на основании закона равенства действия и противодействия возникают равные и противоположно направленные силы взаимодействия.

Сила, с которой связь действует на рассматриваемое тело, называется реакцией связи или просто реакцией.

Сила, с которой тело действует на связь, называется силой давления на связь. Таким образом, сила реакции и сила давления на связь – две равные по модулю силы, имеющие противоположное направление.

Задачи на равновесие несвободных тел решаются в такой последовательности:

1 Выяснить, какое тело (точка) в данной задаче находится в состоянии равновесия, и приложить к нему заданные силы.

2 Выделенное тело освободить от связей и их действие заменить силами реакций.

3 Выбрать направление координатных осей.

4 Составить уравнения равновесия в одном из двух видов:  либо

5 Решить уравнения равновесия.

6 Проверить правильность решения задачи.

Методику решения задач с помощью уравнений равновесия рассмотрим на примерах.

Первую задачу,можно решать после изучения тем 1.3 «Пара сил» и 1.4 «Плоская система произвольно расположенных сил».

Проекция вектора – скалярная величина, которая определяется отрезками, отсекаемыми перпендикулярами, опущенными из начала и конца вектора на ось. Проекция вектора считается положительной (+), если направление ее совпадает с положительным направлением оси (рисунок 1, а), и отрицательной (-), если проекция направлена в противоположную сторону (рисунок 1, б). Если сила перпендикулярна оси, то её проекция равна нулю (рисуное 1,в).

Моментом силы относительно точки называется произведение модуля силы на плечо, т.е. на длину перпендикуляра, восстановленного из точки, относительно которой берется момент, к линии действия силы. Момент принято считать положительным, если он стремится повернуть тело по часовой стрелке (рисунок 2, а), и отрицательным (рисунок 2, б), если вращение направлено в противоположную сторону.

Между моментом пары и моментами сил пары относительно любой точки существует такая важная зависимость: алгебраическая сумма моментов сил пары относительно любой точки – величина постоянная для данной пары и равна ее моменту. Так как пара сил не имеет равнодействующей, ее нельзя уравновесить одной силой.

Момент пары сил в СИ выражается в ньютонометрах (Н×м) или в единицах, кратных ньютонометру: кН×м, МН×м и т.д.

Пример 1. Однородная балка закреплена в точке A с помощью шарнирно-неподвижной опоры и поддерживается в точке B стержнем (рис.3, а). Найти реакции шарнирно-неподвижной опоры и стержня BC. Силой тяжести балки и стержня пренебречь.

Последовательность определения момента силы F относительно оси z (рис.2, в): а) задают модуль и направление силы F; б) указывают координатные оси; в) перпендикулярно оси выбирают плоскость X0Y; г) силу F проецируют на эту плоскость и определяют модуль Fxy этой проекции; д) опустив перпендикуляр из начала координат на проекцию Fxy, определяют плечо a = OK; е) определяют момент Mz(F) = Fxya.

Момент силы относительно оси равен нулю, если сила и ось лежат в одной плоскости: а) сила действует вдоль оси; б) сила F пересекает ось z; в) сила F параллельна оси z.

Решение. Изобразим балку вместе с нагрузками, соблюдая заданные размеры ее участков и угла a (рисунок 3, б).

Освободим балку от связей в точках A и B, заменив эти связи их реакциями. Начало координат поместим в точке A, ось x совместим с осью балки, а ось y направим перпендикулярно балке. Если стержень растягивается, то его реакция направлена в сторону от рассматриваемого тела, а при сжатии – от стержня к телу.

Составим три уравнения равновесия:

 — алгебраическая сумма проекций сил на ось x;

 — алгебраическая сумма проекций сил на ось y;

 — алгебраическая сумма моментов относительно точки A.

Уравнение проекций сил на ось x имеет вид

                                         (1)

Силы F и YA не вошли в уравнение, так как они перпендикулярны оси x и их проекции на эту ось равны нулю.

Проекции силы на ось y

(2)

реакция XA перпендикулярна оси y и ее проекция на эту ось равна нулю.

Для составления уравнения моментов за центр моментов принимаем точку A. Плечо силы RB равно длине перпендикуляра, восстановленного из точки A (центра моментов) к линии действия силы RB. Из рисунка 3, б видно, что .

(3)

Подставив числовые значения, получим

 Н.

Выразим из (2)

Подставив значения сил, получим

 Н.

Из  уравнения (1)  Н.

Проверим правильность решения задачи, составив уравнения моментов относительно точки B:

Подставив числовые значения 3000×4-2000-4000×2,5=0; 12000-12000=0.

Задача решена верно, так как при подстановке получили тождество 0=0.

Вторую задачу контрольной работы решать после изучения темы 1.5 «Центр тяжести». В этих задачах требуется находить центр тяжести плоских фигур, составленных из простых геометрических фигур. Положение центра тяжести плоской фигуры определяется по формулам:  и

где xC и yC – искомые координаты центра тяжести фигуры; xi и yi – координаты центров тяжести составных частей фигуры, которые определяются из заданных размеров; Ai – площади составных частей.

Последовательность решения таких задач рассмотрена в примере.

Пример 2. Вычислить координаты центра тяжести сечения плоской фигуры (рисунок4, а).

Решение. Заданную плоскую фигуру разбиваем на составные части, центры тяжести которых легко определяются (рисунок 4, б) – прямоугольник I, треугольник II и прямоугольники III и IV.

Располагаем координатные оси, как показано на рисунке 4, а.

Находим площади каждой части и координаты xi и yi их центров тяжести/ Все эти данные заносим в таблицу 1.

Таблица 1

Составная часть

Решение: Объект равновесия точка С

Задача 1
Конструкция состоит из двух стержней, соединенных между собой и с основанием шарнирами (рис.1). К шарнирному болту С привязан груз Р. Требуется определить внутренние усилия в стержнях и подобрать их сечение по допускаемым напряжениям на сжатие и растяжение. Величина силы Р, форма сечения и допускаемые напряжения приведены в табл.1.

Рис.1

Таблица1


Вариант

Сечение стержней

Величина Р, кН

, МПа

, МПа



40

220

200

Пример 1. Дано: Р = 10 кН;

Определить реакции в стержнях АС и ВС (рис. 1,а). Подобрать сечение стержней: для стержня ВС в виде трубы с соотношение диаметров d/D=0,8, для стержня АС – квадратное сечение, если допускаемое напряжение на сжатие = 160 МПа, на растяжение =100 МПа.


Рис.1,а Рис.1,б
Решение:

1.Объект равновесия – точка С;

2.“Обрываем” связи заменяя их реакциями этих связей, направленными вдоль стержней (рис. 1,б). При этом предполагаем, что оба стержня растянуты, то есть усилия направлены от точки С.

3.Направляем оси координат Х и У, принимая ось Х перпендикулярно стержню АС.

4.Составляем уравнения равновесия плоской сходящейся системы сил, приложенных к точке С.
Pcos45 – NВСsin30 = 0; откуда
кH.
Усилие получилось положительное, следовательно, стержень ВС работает на растяжение.
Pcos45 + NAC+ NDCcos30 = 0; откуда

-26,3 кH.
Усилие получилось отрицательное, следовательно, стержень АС работает на сжатие.

5. Подбор сечения осуществляем, используя условие прочности при растяжении-сжатии:

,
где N- внутреннее усилие, А – площадь сечения.

Для стержня ВС мм2.

Так как площадь трубы , то .

Для стержня АС мм2.

Так как площадь квадрата , то .
Контрольные вопросы


  1. Как находится проекция силы на ось?

  2. В каком случае проекция силы на ось равна нулю?

  3. В каком случае проекция силы на ось равна модулю силы?

  4. В каком случае проекция силы на ось отрицательна?

  5. Сколько уравнений равновесия составляется для плоской сходящейся системы сил?

  6. В какую сторону направлена реакция стержня с шарнирным крепление концов?

  7. Какие напряжения возникают в стержне ?

  8. Зависят ли напряжения в стержне при его растяжении-сжатии от площади поперечного сечения?

  9. Какой вид имеет условие прочности при растяжении-сжатии?

  10. Как работают хрупкие материалы при растяжении-сжатии?

  11. Как работают пластичные материалы при растяжении-сжатии?

  12. Какие элементы конструкции называются стержнем?

Задача 2
Для заданной схемы балки (рис. 2) требуется определить опорные реакции, построить эпюры изгибающих моментов, найти максимальный момент Мmaxи подобрать стальную балку двутаврового поперечного сечения при [] = 160 МПа. Данные взять из табл. 2.

Рис.2
Таблица 2


Варианты

a, м

b, м

l, м


Изгибаю-щий момент М, кН*м

Сосредо-точенная сила F, кН

2,4

3,6

11

8

18

Пример 2. Для заданной схемы балки (рис. 2,а) требуется написать выражения поперечных сил и изгибающих моментов для каждого участка в общем виде, построить эпюры Q и М, найти максимальный момент Мmaxи подобрать стальную балку двутаврового поперечного сечения при [] = 160 МПа, если L=10м, а=5м, b=2м, М=8 кНм, F=18 кН.


Рис.2,а
Решение. 1. Определяем опорные реакции. Так как горизонтальная нагрузка отсутствует, то опора А имеет только вертикальную реакцию RA. Составляем уравнения равновесия в виде моментов всех сил относительно точек А и В.

откуда находим

,.

Для проверки составим уравнение равновесия на вертикальную ось:

2.Построение эпюр Q и М.

Воспользуемся правилом знаков. Если внешняя сила слева от сечения направлена вверх, то она создает положительную поперечную силу и изгибающий момент. Внешняя сила справа от сечения , направленная вниз создает положительную поперечную силу и отрицательный изгибающий момент.

Если внешний сосредоточенный момент слева от сечения направлен по часовой стрелке, то он создает положительный изгибающий момент. Внешний сосредоточенный момент справа от сечения, направленный против часовой стрелки, создает положительный изгибающий момент.

Рассмотрим первый участок: .

, ,

Рассмотрим второй участок: .

, ,

Рассмотрим третий участок ( идем от правого края): .

, ,

По полученным значениям строим эпюры Q и М (рис.2,а).

После построения эпюр внутренних усилий контролируем их правильность.

На эпюре Q в месте приложения сосредоточенных сил наблюдаются скачки на величину и в направлении этих сил. На эпюре М в месте, где приложен сосредоточенный момент, имеет место скачок на величину и в направление этого момента. Там, где приложена сосредоточенная сила, на эпюре моментов прямая меняет угол наклона.

Эпюру моментов можно построить, не вычисляя закон изменения моментов на каждом участке, а определяя момент в каждой характерной точке. Для указанной балки это точки А,В,С и D.

Момент в точка А : МА=0.

Момент в точке С ( слева от сечения): МС =RA .a= = -17,5 кНм.

Момент в точке С ( справа от сечения): =RA .a= -8= -25,5 кНм.

Момент в точке D: МD=0.

Момент в точке В ( справа от сечения): МВ =-F .b= = -36 кНм.

Момент в точке В ( слева от сечения): МВ =-F .b= = -36 кНм.

3. Подбор сечения.

По эпюре моментов определяем значение максимального изгибающего момента Мmax=36кНм (.Мmax берем по модулю).

Из условия прочности по нормальным напряжениям

,

откуда .

По сортаменту прокатной стали выбираем двутавр №22 с WZ=232 cм3.

Контрольные вопросы
1. Как находится момент силы относительно точки?

2. В каком случае момент силы относительно точки равен нулю?

3. Сколько уравнений равновесия составляется для плоской произвольной системы сил?

4. Как направлены реакции в неподвижном шарнире?

5. Как направлена реакция в подвижном шарнире?

6. Какой элемент конструкции называется балкой?

7. Какие внутренние усилия возникают в балке?

8. Как определяется знак внутреннего изгибающего момента?

9. Какие напряжения возникают в балке?

10. По каким напряжениям производится расчет балок на прочность?

11. Как сформулировать условие прочности при изгибе?

12. Что такое осевой момент сопротивления?

Статьи — Abitu.net

1. Основные определения

Удивительно, но с векторными величинами разной природы (перемещением, скоростью, силой, импульсом и др.) можно работать в значительной мере единообразно — как с геометрическими объектами — геометрическими векторами, или просто векторами, хотя есть и нюансы (см. ниже).

Вектор пред­ставляет собой направленный отрезок прямой, для которого определены правила (законы) сложения с другими векторами, правило вычитания векторов, правило умножения вектора на число, скалярное произведение двух векторов и некоторые другие операции.

Стрелка компаса — не вектор, т. к. для неё нет таких операций.

Мы будем рассматривать векторы на плоскости и в соответствии со сложившейся традицией обозначать их латинскими буквами со стрелками наверху, например: `vec v`, `vec F`, `vec a`, `vec b` и т. п. Часто в целях экономии используют упрощённое обозначение — букву с чертой, например, `bar v` или `bar F`.

Одну из граничных точек вектора называют его началом, а другую — концом. Направление вектора задаётся от начала к концу, причём на чертеже конец вектора отмечают стрелкой. Начало вектора называют также точкой его приложения. Если точка `A` является нача­лом вектора `vec a`, то мы будем говорить, что вектор `vec a` приложен в точке `A` (рис. 2).

Число, выражающее длину направленного отрезка, называют модулем вектора и обозначают той же буквой, что и сам вектор, но без стрелки наверху, например: модулем вектора `vec v` является число `v`. Часто для обозначения модуля вектора прибегают к помощи знака абсолютной величины и пишут, например, `|vec v|` или `|vec F|`.

Вектор называется нулевым, если его начало и конец совпадают. Нулевой вектор не имеет определённого направления и его длина (модуль) равна нулю.

Векторы называются коллинеарными, если они лежат либо на одной прямой, либо на параллельных прямых. Так, например, на рис. 3 векторы `vec a`, `vec b` и `vec c` коллинеарны. 

Два вектора называются равными, если они коллинеарны, имеют одинаковую длину и одинаковое направление.

На рис. 4 слева изображены неравные векторы `vec a` и `vec f`, `vec g` и `vec h`, а справа — равные векторы `vec p` и `vec q`. Точка приложения геометрического вектора `vec a` может быть выбрана произвольно. Мы не различаем двух равных векторов, имеющих разные точки приложения и получающихся один из другого параллельным переносом. В соответствии с этим векторы, изучаемые в геометрии, называют свободными (они определены с точностью до точки приложения).

В физике точка приложения вектора иногда имеет  принципиальное значение. Достаточно вспомнить рычаг: две равные по модулю силы, направленные в одну и ту же сторону, производят на рычаг разное действие, если плечи сил не равны друг другу. И всё же сами силы равны друг другу! Бывают и случаи, когда вектору трудно приписать конкретную точку приложения. Например, если одна система отсчёта движется  относительно другой со скоростью `vec v`, то какой точке  приписать эту скорость?  Всем точкам движущейся системы!

2. Сложение двух векторов.

Пусть даны два произвольных вектора `vec a` и `vec b` (рис. 5а). 

Для нахождения их суммы нужно перенести вектор `vec b` параллельно самому себе так, чтобы его начало совпало с концом вектора `vec a`. Тогда вектор, проведённый из начала вектора `vec a` в конец перенесённого вектора `vec b`, и будет являться суммой `vec a` и `vec b`. На рис. 5б — это вектор `vec c`.

Описанное правило есть просто определение суммы векторов. Как и в случае с числами, сумма векторов не зависит от порядка слагаемых, и поэтому можно записать

`vec c = vec a + vec b = vec b + vec a`.                                                 (1)

Приведённое выше правило геометрического сложения векторов называется правилом треугольника.

Сумма векторов может быть найдена и по правилу параллелограмма. В этом случае параллельным переносом нужно совместить начала векторов `vec a` и `vec b` и построить на них, как на сторонах,  параллелограмм. Тогда сумма `vec a` и `vec b` будет представлять собой диагональ этого параллелограмма, конкретно — суммой `vec a` и `vec b` будет вектор, начало которого совпадает с общим началом векторов `vec a` и `vec b` конец расположен в противоположной вершине параллелограмма, а длина равна длине указанной диагонали (рис. 5в).

Оба способа сложения дают идентичный результат и одинаково часто применяются на практике. Когда речь идёт о нахождении суммы трёх и более векторов, часто последовательно используют  правило  треугольника. Поясним сказанное.

3. Сложение трёх и более векторов. 

Пусть нужно сложить три вектора `vec a`, `vec b` и `vec d` (рис. 6). 

Для этого  по правилу треугольника сначала находится сумма любых двух векторов, например `vec a` и `vec b`, потом полученный вектор `vec c = vec a + vec b` по тому же правилу складывается с третьим  вектором  `vec d`. Тогда  полученный  вектор `vec f = vec c + vec d` и  будет представлять собой сумму  трёх  векторов `vec a`, `vec b` и `vec d`: `vec f = vec a + vec b + vec d`. Как и в случае с двумя векторами, порядок слагаемых не влияет на конечный результат.

Чтобы упростить процесс сложения трёх и более векторов, обычно не находят промежуточные суммы типа `vec c = vec a + vec b`, а применяют правило многоугольника: параллельными переносами из конца первого вектора откладывают второй, из конца второго — откладывают третий, из конца третьего  — четвёртый  и  т.@`, хотя складывать температуры (числа) никто не запрещает. Всё же чаще всего сумма температур представляет собой никому не нужную величину; она редко входит в какие-либо уравнения (входит почти случайно).

Иное дело – с массой. Если система состоит из тел с массами `m_1`, `m_2`, `m_3` и т. д., то масса всей системы равна `m = m_1 + m_2 + m_3 + ` и т. д. (Если на лифте написано, что максимальный груз, перевозимый лифтом, равен `500` кг, то перед входом в лифт нужно убедиться, что сумма масс вносимых в лифт грузов не превышает `500` кг.) Говорят, что масса – есть аддитивная величина (от английского слова add – добавлять, прибавлять, складывать). А вот температура – не аддитивная величина.

Сила есть аддитивная векторная величина. Если к телу в точке (или к системе тел в разных точках!) приложены силы `vec(F_1)`, `vec(F_2)`, `vec(F_3)` и т. д., то сумма векторов сил `vec(F_1) + vec(F_2) + vec(F_3) + …` есть осмысленная и даже очень нужная величина. Например, в условиях равновесия тела сумма всех приложенных к нему сил `vec(F_1) + vec(F_2) + vec(F_3) + … = 0`, даже если силы приложены в разных точках тела. Причём это относится не только к твёрдым телам. Если нитка подвешена за два конца к двум гвоздям, а в промежутке перекинута еще через какие-нибудь гвозди, то сначала нужно найти силы со стороны каждого из гвоздей и  силу со стороны Земли (силу тяжести) `vec F_1`, `vec(F_2)`, `vec(F_3)`, `…`; при этом говорят, что к нитке приложена сумма сил `vec(F_1) + vec(F_2) + vec(F_3) + …`; в условиях равновесия эта сумма будет равна нулю.

Не так со скоростями. Если система состоит из двух частиц, имеющих в некоторый момент времени скорости `vec(v_1)` и `vec(v_2)`, то это не означает, что в этот момент вся система обладает скоростью равной векторной сумме `vec(v_1) + vec(v_2)`. Никто не запрещает складывать векторы скорости разных частиц; но с точки зрения физики вектор `vec(v_1) + vec(v_2)` ничему приписать нельзя. В этом смысле скорость — не аддитивная величина. Суммой скоростей (векторной суммой) интересуются, когда одно движение накладывается на другое (например, Земля вращается вокруг Солнца, но вместе с Солнцем движется вокруг центра Галактики). А вот сумма скоростей отдельных частиц системы (например, сумма скоростей звезд в Галактике) физического интереса не представляет.

Родственная скорости величина, с которой вы еще не раз встретитесь в курсе физики, импульс материальной точки, равный произведению массы на скорость, `vec p = m vec v` снова — величина аддитивная.

В последнем равенстве мы встречаемся с умножением вектора на скаляр. Поясним эту процедуру.

4. Умножение вектора на скаляр. 

Произведением вектора `vec a` на число `k` называют новый вектор `vec b = k vec a`, коллинеарный вектору `vec a`, направленный в ту же сторону, что и вектор `vec a`, если `k > 0`, и в противоположную сторону, если `k < 0`, а модуль `b` равен

 `b = |k| a`                                                                                (2)

где `|k|` — абсолютная величина числа `k`. 

Если два вектора коллинеарны, то они отличаются только скалярным множителем. Наоборот, если два вектора отличаются только ска­лярным множителем, не равным  нулю, то они коллинеарны.      

В случае, когда `k = 0` или `vec a = 0`, произведение `k vec a` представляет собой нулевой  вектор,  направление которого не определено.

Если `k = 1`, то согласно (2) `vec b = vec a` и векторы `vec a` и `vec b` равны (рис. 8а).

При `k = — 1` получим `vec b = — vec a`. Вектор `- vec a` имеет модуль, равный модулю вектора `vec a`, но направлен в противоположную сторону (рис. 8б).

Два  вектора,  противоположно  направленные и имеющие  равные длины, называются противоположными.

Импульс тела `vec p = m vec v` коллинеарен вектору скорости и направлен с ней в одну сторону, т. к. массы всех тел положительны. Чуть ранее говорилось об аддитивности импульса. Если система состоит из материальных точек с массами `m_1`, `m_2`, `m_3`, `…`, которые в некоторый момент времени имели скорости `vec(v_1)`, `vec(v_2)`, `vec(v_3)`, `…`, т. е. имели импульсы `vec(p_1) = m_1 vec(v_1)`, `vec(p_2) = m_2 vec(v_2)`, `vec(p_3) = m_3 vec(v_3)`, `…`, то вся система в этот момент обладает импульсом  

`vec p = vec(p_1) + vec(p_2) + vec(p_3) + … = m_1 vec(v_1) + m_2 vec(v_2) + m_3 vec(v_3) + …`.

При этом каждое из слагаемых здесь должно быть найдено по правилу умножения вектора (скорости данной частицы) на скаляр (её массу), а затем все эти векторы должны быть сложены, например, по правилу многоугольника.

5. Разность двух векторов.

Вычесть из вектора `vec a` вектор `vec b` означает прибавить к вектору `vec a` вектор   `- vec b`:

`vec a — vec b = vec a + (- vec b)`;

см. рис.  9а, 9б.

Плоская система сходящихся сил — Студопедия.Нет

Студент должен:

 иметь представление:

—о плоской системе сходящихся сил;

—о приведении сил к одной точке;

—о равнодействующей силе;

—о равновесии системы сил.
знать:

—геометрический и аналитический способы определения равнодействующей силы;

—условия равновесия системы сил;
уметь:

—определять равнодействующую системы сил;

—решать задачи на равновесие системы сил в аналитической форме, рационально выбирая направление координатных осей.

Система сходящихся сил. Способы сложения двух сил. Разложение силы на две составляющие. Определение равнодействующей системы сил геометрическим способом. Силовой многоугольник. Условие равновесия в векторной форме.

Проекция силы на ось, правило знаков. Проекция силы на две взаимоперпендикулярные оси. Аналитическое определение равнодействующей. Условие равновесия в аналитической форме. Рациональный выбор координат осей.

Литература 1, с. 21…28; 3, с. 36…59

Практические занятия №1

Методические указания

Силы называются сходящимися, если их линии действия пересекаются в одной точке. Различают плоскую систему сходящихся сил, когда линия действия всех данных сил лежит в одной плоскости, и пространственную систему сходящихся сил, когда линии действия сил лежат в разных плоскостях.

Эта система эквивалентна одной силе (равнодействующей) и стремиться придать телу (в случае если точка схода всех сил совпадает с центром тяжести тела) прямолинейное движение. Равновесие тела будет иметь место в случае равенства равнодействующей нулю. Геометрическим условием равновесия является замкнутость многоугольника, построенного из сил системы, аналитическим условием — равенство нулю алгебраических сумм проекций сил на любые две взаимно перпендикулярные оси. Следует получить твердые навыки в решении задач на равновесие тел, обратив особое внимание на рациональный выбор направления координатных осей.

 

Вопросы для самоконтроля

1.Как геометрически находится равнодействующая плоской системы сходящихся сил?

2.Что называется проекцией силы на ось? В каком случае проекция силы на ось равна модулю силы? В каком случае проекция силы на ось равна нулю?

3.Как найти числовое значение и направление равнодействующей системы сил, если заданы проекции составляющих сил на две взаимно перпендикулярные оси?

4.Сформулируйте аналитическое условие равновесия плоской системы сходящихся сил.

 

Тема 1.3 Пара сил и момент силы относительно точки

Студент должен:

иметь представление:

—о силах, создающих пару, и действии, оказываемом ими на тело;

—о моменте силы относительно точки и действии ее на тело;
знать:

—момент пары сил: обозначение, модуль, знак;

—свойства пар сил;

—момент силы относительно точки; модуль, знак, обозначение;

—частные случаи;
уметь:

—определять момент пары сил и результирующей пары системы пар сил;

—рассчитывать момент- силы относительно точки.

Сложение двух параллельных сил. Пара сил и ее характеристики. Момент пары. Эквивалентные пары. Сложение пар. Условие равновесия системы пар сил. Момент силы относительно точки.

Литература 1, с. 28…45; 3, с. 67…75

 

Методические указания

Две равные и параллельные силы, направленные в противоположные стороны и не лежащие на одной прямой, называются парой сил. Примером такой системы сил могут служить усилия, передаваемые руками шофера на рулевое колесо автомобиля. Пара сил имеет очень большое значение в практике. Именно поэтому свойства пары как специфической меры механического взаимодействия тел изучается отдельно. Сумма проекций сил пары на любую ось равна нулю, т. е. пара сил не имеет равнодействующей. Несмотря на это тело под действием пары сил но находится в равновесии.

Система пар сил эквивалентна одной паре (равнодействующей) и стремится придать телу вращательное движение. Равновесие тела будет иметь место в случае равенства нулю момента равнодействующей пары. Аналитическим условием равновесия является равенство нулю алгебраической суммы моментов пар системы. Следует обратить особое внимание на определение момента силы относительно точки, а также оси. Необходимо помнить, что момент силы относительно точки равен нулю лишь в случае, если точка лежит на линии действия силы, а относительно оси — лишь тогда, когда сила и ось лежат в одной плоскости (т. е. линия действия силы или параллельна оси, или пересекает ее).

Вопросы для самоконтроля

1.Что такое пара сил?

2.Что такое момент пары сил, плечо пары сил?

3.Назовите свойства пар сил.

4.Сформулируйте условие равновесия системы пар сил.

Пространственная система сил. Теорема Вариньона.

Пространственная система сил



Пространственная система сходящихся сил

Система сил, линии действия которых расположены в различных плоскостях, называется пространственной системой сил.

Пространственная система сил называется сходящейся, если линии действия всех сил системы пересекаются в одной точке.

Теорема: пространственная система сходящихся сил эквивалентна равнодействующей, которая равна векторной сумме этих сил; линия действия равнодействующей проходит через точку пересечения линий действия составляющих сил.

Пусть дана пространственная система n сходящихся сил (F1, F2, F3,….Fn). На основании следствия из аксиом III и IV перенесем все силы системы вдоль линий действия в точку их пересечения. Затем на основании аксиомы параллелограмма последовательно сложим все силы и получим их равнодействующую:

FΣ = F1 + F2 + F3 + ….+ Fn,    или    FΣ = ΣFi.

Силовой многоугольник пространственной системы сил не лежит в одной плоскости, поэтому геометрический и графический способы нахождения равнодействующей пространственной системы сходящихся сил неприемлемы, а применяется только аналитический способ (метод проекций).

Проекция силы на ось в пространстве находится по проецирующим перпендикулярам, и может быть определена при помощи тригонометрических функций. При определении проекций сил пространственной системы потребуется система координат с осями X, Y, Z, поскольку силы системы не располагаются в одной плоскости.

Правило знаков для проекций будет таким же, как и для плоской системы сил – совпадающие по направлению с координатной осью силы считаются положительными, в противном случае – отрицательными. Если вектор силы параллелен какой-либо оси координат, то он проецируется на эту ось в натуральную величину, если же вектор перпендикулярен оси, его проекция на эту ось будет равна нулю.

***

Разложение силы по трем осям координат

Пусть дана сила F (см. рисунок 1).
Возьмем систему координат так, чтобы начало координат совпадало с началом вектора силы F (т. е. с точкой приложения силы). Из конца этого вектора опустим перпендикуляр на плоскость xy и разложим силу F на составляющие Fxy и Fz, а составляющую Fxy – на составляющие Fx и Fy. Тогда:

F = Fx + Fy + Fz.

Достроим полученное изображение до параллелепипеда, у которого составляющие Fx, Fy и Fz являются ребрами, а сила F – диагональю.

Из изложенного можно сделать вывод: равнодействующая трех взаимно-перпендикулярных сил выражается по модулю и направлению диагональю параллелепипеда, построенного на этих силах.

Из рисунка видно, что в случаях разложения силы F по трем взаимно-перпендикулярным направлениям x, y, z составляющие Fx, Fy и Fz равны по модулю проекциям силы F на эти оси.

Зная проекции силы на три взаимно-перпендикулярные оси координат, можно определить модуль и направление вектора силы по формулам:

модуль силы:   F = √(Fx2 + Fy2 + Fz2)    (здесь и далее √ — знак корня);

направляющие косинусы:   cos(F,x) = Fx/F;    cos(F,y) = Fy/F;    cos(F,z) = Fz/F.

***

Аналитический способ определения равнодействующей пространственной системы сходящихся сил

Рассмотренный выше способ разложения силы F на три составляющие по направлению координатных осей x, y, z можно применить для каждой из сходящихся сил пространственной системы. Тогда вместо данной системы n сходящихся сил мы получим эквивалентную ей систему 3n сил, из которых n сил действуют по оси x, n сил – по оси y, и n сил – по оси z.
Равнодействующая проекций сил системы на ось x равна их геометрической сумме, то же самое можно сказать и о равнодействующих проекций сил на оси y и z.
Таким образом, систему 3n сил можно заменить эквивалентной ей системой трех сил, каждая из которых представляет собой равнодействующую проекций сил данной системы на ту или иную ось координат.

Проекции силы на три взаимно-перпендикулярные оси и составляющие силы, направленные по этим осям, равны по модулю, следовательно, проекции равнодействующей равны:

FΣx = ΣX;     FΣy = ΣY;     FΣz = ΣZ.

Очевидно, что равнодействующая трех взаимно перпендикулярных сил выражается по модулю и направлению диагональю параллелепипеда, построенного на этих силах, и по известным проекциям равнодействующей можно определить модуль и направление самой равнодействующей.

***

Аналитические условия равновесия пространственной системы сходящихся сил

Известно, что пространственная система сходящихся сил эквивалентна равнодействующей. Если такая система сил находится в равновесии, т. е. эквивалентна нулю, то можно сделать вывод, что равнодействующая этой системы равна нулю, а следовательно, и проекции равнодействующей тоже равны нулю, причем эти проекции равны сумме проекций составляющих.
Отсюда вытекают условия равновесия пространственной системы сходящихся сил:

ΣX = 0;    ΣY = 0;    ΣZ = 0.

Эти условия формируются следующим образом: для равновесия пространственной системы сходящихся сил необходимо и достаточно, чтобы алгебраическая сумма проекций всех сил на каждую их трех координатных осей равнялась нулю.

***



Момент силы относительно оси

Рассмотрим колесо червячной передачи, укрепленное на валу, вращающемся в подшипниках (см. рисунок 2). Червяк передает червячному колесу силу F, не лежащую в плоскости, перпендикулярной оси.

Разложим силу F на три взаимно-перпендикулярные составляющие F1, F2 и F3.
Составляющую F1 назовем окружной силой, составляющую F2 – осевой силой, а составляющую F3 – радиальной силой.
Из рисунка видно, что составляющая F1 вызывает вращательное действие, которое измеряется произведением силы F1 на радиус колеса r; составляющая F2 стремится сдвинуть червячное колесо вдоль оси, а составляющая F3 стремится изогнуть ось колеса.
Очевидно, что вращающее действие сил F2 и F3 относительно оси колеса равно нулю.
Таким образом, если нужно найти момент силы относительно оси, то следует принимать в расчет только составляющую F1, лежащую в плоскости, перпендикулярной оси, и не пересекающую ось (иначе ее момент будет равен нулю).

Ранее было отмечено, что проекция вектора силы на ось есть скалярная алгебраическая величина. В отличие от проекции на ось проекция силы на плоскость есть величина векторная, так как эта проекция характеризуется не только числовым значением, но и положением на плоскости, т. е. направлением.
Поэтому моменту силы относительно оси можно дать такое определение: моментом силы относительно оси называется величина, равная моменту проекции этой силы на плоскость, перпендикулярную оси, относительно точки пересечения оси с плоскостью.

Это определение поясняет рисунок 3.
Момент силы относительно оси условимся записывать следующим образом:

Mz(F) = Fна.

Условимся считать момент силы положительным, если смотреть с положительного конца оси и сила стремится вызвать вращение против часовой стрелки, если же сила стремится вызвать вращение по часовой стрелке, ее момент считаем отрицательным.

Момент силы относительно оси не меняется при перемещении силы вдоль оси ее действия.

Момент силы будет равен нулю в двух случаях (не считая случаев, когда сила равна нулю или направлена вдоль оси):
  • если вектор силы параллелен оси, так как при этом проекция силы на плоскость, перпендикулярную оси, равна нулю (см. рисунок 3, сила FZ);
  • если линия действия силы пересекает ось, так как при этом плечо равно нулю (сила F3 на рисунке 2).

***

Аналитические условия равновесия пространственной системы произвольно расположенных сил

Пространственная система сил, в которой линии действия составляющих сил расположены произвольно, т. е. линии их действия могут не пересекаться и находиться в разных плоскостях, называется произвольно расположенной системой сил.

Для равновесия пространственной системы произвольно расположенных сил необходимо и достаточно, чтобы алгебраическая сумма проекций всех сил на каждую из трех осей координат была равна нулю и чтобы алгебраическая сумма моментов всех сил относительно каждой из этих осей была равна нулю.

Строгое обоснование приведенного выше условия равновесия пространственной системы произвольно расположенных сил требует знания некоторых вопросов, не предусмотренных программами учреждений среднего профессионального образования, поэтому условие равновесия такой системы здесь приводится без доказательства.

Математически условие равновесия пространственной системы произвольно расположенных сил можно записать в виде уравнений:

  • ΣX = 0;     ΣMx(Fi) = 0;
  • ΣY = 0;     ΣMy(Fi) = 0;
  • ΣZ = 0;     ΣMz(Fi) = 0.

Свободное тело в пространстве имеет шесть степеней свободы, а именно: возможность перемещаться в направлениях трех взаимно-перпендикулярных осей координат и возможность вращаться вокруг этих осей. Таким образом, шести степеням свободы тела в пространстве соответствуют шесть условий равновесия.
Если система сил, приложенных к свободному телу, удовлетворяет всем шести условиям равновесия, то возможность трех перемещений и трех вращений тела под действием сил системы исключена, поэтому тело будет находится в равновесии.

Очевидно, что все выведенные ранее условия равновесия для различных систем сил являются частными случаями условия равновесия пространственной системы произвольно расположенных сил.

Так как условия равновесия пространственной системы сил справедливы для любых прямоугольных осей координат, то при решении данной задачи систему координат можно изменять, т. е. часть уравнений равновесия составить для одних осей координат, а часть – для измененных. В некоторых случаях этот прием упрощает решение задач.

***

Теорема о моменте равнодействующей относительно оси


(теорема Вариньона)

Теорема: момент равнодействующей относительно оси равен алгебраической сумме моментов, составляющих сил относительно этой же оси.

Пусть даны пространственная система n произвольно расположенных сил, приложенных к телу, и равнодействующая этой системы сил FΣ (см. рисунок 4):

(F1, F2, F3,….Fn) ≡ FΣ.

Приложим к телу другую систему сил, равнодействующая которой F’Σ по модулю равна FΣ и направлена по той же линии действия, но в противоположную сторону, т. е. является уравновешивающей данной системы сил.
Тогда можно записать:

(F1, F2, F3,….Fn, F’Σ) ≡ 0 ,   или   (FΣ, F’Σ) ≡ 0.

Так как обе записанные выше системы сил эквивалентны нулю, т. е. уравновешены, то к ним можно применить любое условие равновесия, например

ΣMx(Fi) = 0.

Запишем это условие для обеих систем:

Mx(F1) = Mx(F2) + Mx(F2) + …. + Mx(Fn) + Mx(F’Σ) = 0;
MΣ(FΣ) + Mx(F’Σ) = 0.

Так как правые части этих равенств равны, то будут равны и левые :

Mx(F1) = Mx(F2) + Mx(F3) + …. +Mx(Fn) + Mx(F’Σ) = Mx(FΣ) + Mx(F’Σ).

Сократив общее слагаемое Mx(F’Σ), получим:

Mx(F1) = Mx(F2) + Mx(F3) + …. +Mx(Fn) = Mx(FΣ)   или   ΣMx(Fi) + Mx(FΣ).

Теорема доказана.

***

Трение — сущность явления, законы и зависимости


Главная страница


Дистанционное образование

Специальности

Учебные дисциплины

Олимпиады и тесты

Памятка по проекциям вектора на ось

Здравствуй, дорогой друг! Вот и наступил 11-й класс. Уже совсем скоро начнутся экзамены, а такие слова, как «сила», «проекция на ось», «векторная сумма сил», ставят тебя в тупик… Ну ничего, сейчас я постараюсь помочь тебе разобраться со всеми этими страшными терминами. Погнали!

Итак, первое, с чего нужно начать, это понять, что же от нас хотят. Ведь распространённой ошибкой, которую допускают многие одиннадцатиклассники, является невнимательное чтение вопроса задачи. «ПТУ или СПБ» … нет, что-то не! «Модуль или проекция»!!! Да, да, да… невнимательно прочтённое задание, а вслед за ним, возможно, упущенный «минус» отделяют потенциального инженера-конструктора от заветного бюджетного места в топовом вузе.

Если от нас требуется найти «модуль силы» (в особо изысканных задачах «модуль проекции силы»), то всё не так страшно. Но если же на твою нелёгкую долю, друг, выпала участь отыскать «проекцию» (или «проекцию модуля»), то будь внимательней самого внимательного внимателя.

Итак:

В первом случае мы видим, что вектор силы F параллелен оси x. Более того, направление вектора совпадает с направлением оси. Тогда проекция вектора равна самому вектору. Во втором случае мы видим, что вектор силы параллелен оси, но его направление уже противоположно. Тогда можно сказать, что проекция также равна самому вектору, но взятому со знаком «минус».

И самый трудный случай, когда вектор даёт проекции сразу на обе оси:

Тут необходимо учитывать угол отклонения вектора от оси. В данном примере взят угол α между самим вектором и осью x. В таком случае проекция вектора на ось x будет равна произведению самого вектора на косинус угла отклонения:

а соответственно, проекция на осьy равна произведению самого вектора на синус угла отклонения:

Также следует внимательно следить за тем, какой угол тебе дан, ведь если взять противоположный угол, то и тригонометрические функции в проекциях поменяются на противоположные.

Надеюсь, прочитав эту маленькую статью, ты стал еще на шаг ближе к своей будущей профессии. Ну а если ты хочешь не просто шагать, а бежать навстречу знаниям, то ждём тебя на занятиях в наших центрах!


Проекции на оси координат. Векторная проекция

Физика для 9 класса (Кикоин И.К., Кикоин А.К., 1999),
задание №5
к главе « ГЛАВА 1. ОБЩАЯ ИНФОРМАЦИЯ О ДВИЖЕНИИ ».

1. Что называется проекцией вектора на координатную ось?

1. Проекция вектора a на ось координат — это длина отрезка между проекциями начала и конца вектора a (перпендикуляры, опущенные из этих точек на ось) на эту ось координат.

2. Как вектор перемещения тела связан с его координатами?

2. Проекции вектора смещения s на оси координат равны изменению соответствующих координат тела.

3. Если координата точки увеличивается со временем, какой знак имеет проекция вектора смещения на координатную ось? А если он уменьшится?

3. Если координата точки увеличивается со временем, то проекция вектора смещения на координатную ось будет положительной, так как в этом случае мы перейдем от проекции начала к проекции конца вектор в направлении самой оси.

Если координата точки уменьшается со временем, то проекция вектора смещения на координатную ось будет отрицательной, так как в этом случае мы перейдем от проекции начала к проекции конца вектора относительно направление самой оси.

4. Если вектор смещения параллелен оси X, то каков модуль проекции вектора на эту ось? А модуль проекции того же вектора на ось Y?

4.Если вектор смещения параллелен оси X, то модуль проекции вектора на эту ось равен модулю самого вектора, а его проекция на ось Y равна нулю.

5. Определите знаки проекций на ось X векторов смещения, показанных на рисунке 22. Как меняются координаты тела во время этих смещений?

5. Во всех следующих случаях координата Y тела не изменяется, но координата X тела изменяется следующим образом:

а) с 1;

проекция вектора s 1 на ось X отрицательна и равна по модулю длине вектора s 1.При таком смещении координата X тела уменьшится на длину вектора s 1.

б) с 2;

проекция вектора s 2 на ось X положительна и равна по модулю длине вектора s 1. При таком смещении координата X тела увеличится на длину вектора s 2.

в) с 3;

проекция вектора s 3 на ось X отрицательна и равна по модулю длине вектора s 3.При таком смещении координата X тела уменьшится на длину вектора s 3.

д) с 4;

проекция вектора s 4 на ось X положительна и равна по модулю длине вектора s 4. При таком смещении координата X тела увеличится на длину вектора s 4

e) s 5;

проекция вектора s 5 на ось X отрицательна и равна по модулю длине вектора s 5.При таком смещении координата X тела уменьшится на длину вектора s 5.

6. Если пройденное расстояние велико, может ли быть малым модуль смещения?

6. Может быть. Это связано с тем, что смещение (вектор смещения) является векторной величиной, т.е. представляет собой направленный отрезок прямой, соединяющий исходное положение тела с его последующими положениями. И конечное положение тела (независимо от величины пройденного расстояния) может быть настолько близко к исходному положению тела, насколько вам нравится.Если конечное и начальное положения тела совпадают, модуль смещения будет равен нулю.

7. Почему в механике вектор движения тела важнее, чем пройденный им путь?

7. Основная задача механиков — определить положение тела в любой момент. Зная вектор движения тела, мы можем определить координаты тела, то есть положение тела в любой момент времени, а зная только пройденное расстояние, мы не можем определить координаты тела, так как мы не иметь информацию о направлении движения, но может судить только о длине пройденного пути в данный момент времени.

Во-первых, давайте вспомним, что это координатная ось , двухкоординатная проекция и координаты точки на оси .

Координатная ось — это прямая линия, которой задано какое-то направление. Вы можете думать об этом как о векторе с бесконечно большим модулем.

Координатная ось обозначается любой буквой: X, Y, Z, s, t … Обычно на оси выбирается (произвольно) точка, которая называется началом координат и, как правило, обозначается буква О.От этой точки отсчитываются расстояния до других интересных нам мест.

Проекция точки на ось — это основание перпендикуляра, опущенного из этой точки на заданную ось (рис. 8). То есть проекция точки на ось есть точка.

Координата точки на ось — это число, абсолютное значение которого равно длине сегмента оси (в выбранном масштабе), заключенного между началом оси и проекцией точки на эту ось.Это число принимается со знаком плюс, если проекция точки расположена в направлении оси от ее начала, и со знаком минус, если в противоположном направлении.

Скалярная проекция вектора на ось — это число , абсолютное значение которого равно длине сегмента оси (в выбранном масштабе), заключенного между проекциями начальной точки и конечная точка вектора. Важный! Обычно вместо выражения скалярной проекции вектора на ось они просто говорят — проекция вектора на ось , то есть слово скаляр опускается. Проекция вектора обозначается той же буквой, что и проецируемый вектор (в обычных, не полужирных обозначениях), с нижним индексом (как правило) имени оси, на которую этот вектор проецируется. Например, если вектор проецируется на ось X a, , то его проекция обозначается x. При проецировании того же вектора на другую ось, скажем, ось Y, его проекция будет обозначаться буквой y (рис. 9).

Для расчета проекции вектора на ось (например, ось X) необходимо вычесть координату начальной точки из координаты ее конечной точки, то есть

а х = х к — х н.

Мы должны помнить: скалярная проекция вектора на ось (или просто проекция вектора на ось) — это число (а не вектор)! Более того, проекция может быть положительной, если значение x k больше значения x n, отрицательной, если значение x k меньше значения x n, и равным нулю, если x k равно x n (рис. 10).

Проекцию вектора на ось также можно найти, зная модуль вектора и угол, который он составляет с этой осью.

На рисунке 11 показано, что ax = a Cos α

То есть проекция вектора на ось равна произведению модуля вектора на косинус угла между направлением оси и направление вектора … Если угол острый, то Cos α> 0 и ax> 0, а если он тупой, то косинус тупого угла отрицательный, и проекция вектора на ось будет также быть отрицательным.

Углы, отсчитываемые от оси против часовой стрелки, считаются положительными, а попутные — отрицательными.Однако, поскольку косинус является четной функцией, то есть Cos α = Cos (- α), то при вычислении проекций углы можно считать как по часовой, так и против часовой стрелки.

При решении задач часто используются следующие свойства проекций: если

a = b + c +… + d , то ax = bx + cx +… + dx (аналогично для другие оси),

a = m b , затем ax = mb x (аналогично для других осей).

Формула a x = a Cos α будет Часто встречаются при решении задач, поэтому вам обязательно нужно знать это. Правило определения проекции нужно знать наизусть!

Помните!

Чтобы найти проекцию вектора на ось, модуль этого вектора должен быть умножен на косинус угла между направлением оси и направлением вектора.

Еще раз — ПО СТРОКЕ!

Ответ:

Свойства проекции:

Свойства векторной проекции

Имущество 1.

Проекция суммы двух векторов на ось равна сумме проекций векторов на одну и ту же ось:

Это свойство позволяет заменить проекцию суммы векторов суммой их проекций и наоборот.

Свойство 2. Если вектор умножается на число λ, то его проекция на ось также умножается на это число:

Имущество 3.

Проекция вектора на ось l равна произведению модуля вектора на косинус угла между вектором и осью:

Орт оси.Разложение вектора в единицах координат. Координаты вектора. Координаты свойств

Ответ:

Единичные векторы осей.

Прямоугольная система координат (любой размерности) также описывается набором единичных векторов, сонаправленных с осями координат. Количество единичных векторов равно размеру системы координат, и все они перпендикулярны друг другу.

В трехмерном случае единичные векторы обычно обозначают

AND Также могут применяться стрелки и обозначения стрелок.

В этом случае в случае правой системы координат верны следующие формулы с векторными произведениями единичных векторов:

Разложение вектора в единицах координат.

Единичный вектор оси координат обозначен сквозным, оси — сквозным, оси — сквозным (рис. 1)

Для любого вектора, лежащего в плоскости, имеет место следующее разложение:

Если вектор расположен в пространстве, то разложение по единичным векторам осей координат имеет вид:

Координаты вектора:

Чтобы вычислить координаты вектора, зная координаты (x1; y1) его начала A и координаты (x2; y2) его конца B, вам необходимо вычесть координаты начала координат из координат конца: (x2 — x1; y2 — y1).

Свойства координат.

Рассмотрим координатную линию с началом в точке O и единичным вектором i. Тогда для любого вектора a на этой прямой: a = axi.

Число ax называется координатой вектора a на координатной оси.

Свойство 1. При добавлении векторов на оси складываются их координаты.

Свойство 2. Когда вектор умножается на число, его координата умножается на это число.

Точечное произведение векторов. Характеристики.

Ответ:

Скалярное произведение двух ненулевых векторов — это число

, равное произведению этих векторов на косинус угла между ними.

Недвижимость:

1. Скалярное произведение имеет движимое свойство: ab = bа

Скалярное произведение единичных векторов координат. Определение скалярного произведения векторов по их координатам.

Ответ:

Единичные векторы точечного произведения (×)

Определение скалярного произведения векторов по их координатам.

Скалярное произведение двух векторов и их координаты можно вычислить по формуле

Произведение двух векторов. Векторные свойства продукта.

Ответ:

Три некопланарных вектора образуют правую тройку, если от конца третьего вращение от первого вектора ко второму выполняется против часовой стрелки.Если по часовой, то налево., Если нет, то наоборот ( покажите, как он показал «ручками»)

Векторное произведение вектора a на вектор b называется вектором , из которого:

1. Перпендикулярно векторам a и b

2. Имеет длину, численно равную площади параллелограмма, образованного на a и b векторах

3. Векторы a, b и c образуют правую тройку векторов

Недвижимость:

1.

3.

4.

Векторное произведение единичных векторов координат. Определение векторного произведения векторов по их координатам.

Ответ:

Векторное произведение единичных векторов координат.

Определение векторного произведения векторов по их координатам.

Пусть векторы a = (x1; y1; z1) и b = (x2; y2; z2) заданы их координатами в прямоугольной декартовой системе координат O, i, j, k и тройке i, j, k правша.

Разложим a и b в базисные векторы:

А знак равно Икс 1 Я + У 1 J + Z 1 К, б = Икс 2 Я + Y 2 J + Z 2 К.

Используя свойства векторного произведения, получаем

[а; b] = =

= x 1 x 2 + x 1 y 2 + x 1 z 2 +

+ y 1 x 2 + y 1 y 2 + y 1 z 2 +

+ z 1 x 2 + z 1 y 2 + z 1 z 2. (1)

По определению векторного произведения находим

= 0, = k, = — j,

= — k, = 0, = i,

= j, = — i.= 0.

С учетом этих равенств формулу (1) можно записать в следующем виде:

[а; b] = x 1 y 2 k — x 1 z 2 j — y 1 x 2 k + y 1 z 2 i + z 1 x 2 j — z 1 y 2 i

[а; b] = (y 1 z 2 — z 1 y 2) i + (z 1 x 2 — x 1 z 2) j + (x 1 y 2 — y 1 x 2) k. (2)

Формула (2) дает выражение для векторного произведения двух векторов, заданных их координатами.

Полученная формула громоздка; используя обозначения определителей, можно записать его в другой форме, более удобной для запоминания:

Обычно формулу (3) пишут еще короче:

Решение задач о равновесии сходящихся сил путем построения замкнутых степенных многоугольников связано с громоздкими конструкциями.Универсальным методом решения таких задач является переход к определению проекций заданных сил на оси координат и оперирование этими проекциями. Осью называется прямая линия, которой приписывается определенное направление.

Проекция вектора на ось — это скалярная величина, которая определяется отрезком оси, отсеченным перпендикулярами, падающими на него из начала и конца вектора.

Векторная проекция считается положительной, если направление от начала проекции до ее конца совпадает с положительным направлением оси.Проекция вектора считается отрицательной, если направление от начала проекции до его конца противоположно положительному направлению оси.

Таким образом, проекция силы на координатную ось равна произведению модуля силы на косинус угла между вектором силы и положительным направлением оси.

Рассмотрим ряд случаев проецирования сил на ось:

Вектор силы F (рис.15) составляет острый угол с положительным направлением оси x.

Чтобы найти проекцию, от начала и до конца вектора силы опускаем перпендикуляры к оси oh ; получаем

1. F x = F cos α

Проекция вектора в данном случае положительная

Сила F (рис. 16) с положительным направлением оси NS тупым углом α.

Тогда F x = F cos α, но поскольку α = 180 0 — φ,

F x = F cos α = F cos180 0 — φ = — F cos φ.

Проекция силы F на ось oh в данном случае отрицательная.

Сила F (рис.17) перпендикулярно оси oh .

Проекция силы F на ось NS равна нулю

F x = F cos 90 ° = 0.

Сила, расположенная на плоскости hou (рис. 18), может проецироваться на две оси координат Oh и OU .

Strength F можно разложить на составляющие: F x и F y. Модуль вектора F x равен проекции вектора F на ось ox , а модуль вектора F y равен проекции вектора F на ось oy .

Из Δ OAV : F x = F cos α, F x = F sin α.

Из Δ SLA : F x = F cos φ, F x = F sin φ.

Модуль силы можно найти по теореме Пифагора:

Проекция векторной суммы или результирующей на любую ось равна алгебраической сумме проекций членов векторов на ту же ось.

Рассмотрим сходящиеся силы F 1, F 2, F 3 и F 4, (рис.19, а). Геометрическая сумма или равнодействующая этих сил F , определяемая закрывающей стороной многоугольника

сил.

Спустимся от вершин многоугольника мощности к перпендикулярам оси x .

Учитывая полученные проекции сил непосредственно от выполненной конструкции, имеем

F = F 1x + F 2x + F 3x + F 4x

, где n — количество членов вектора.Их проекции включают указанное выше уравнение с соответствующим знаком.

В плоскости геометрическую сумму сил можно спроецировать на две оси координат, а в пространстве, соответственно, на три.

В этой статье мы рассмотрим проекцию вектора на ось и узнаем, как найти числовую проекцию вектора. Сначала мы определим проекцию вектора на ось, введем обозначения, а также дадим графическую иллюстрацию. После этого мы объявим определение числовой проекции вектора на ось, рассмотрим, как его найти, и покажем решения нескольких примеров, в которых требуется найти числовую проекцию вектора на ось.

Навигация по страницам.

Проекция вектора на ось — определение, обозначения, иллюстрации, пример.

Начнем с общей информации.

Под осью понимается прямая линия, для которой указано направление. Таким образом, проекция вектора на ось и проекция вектора на направленную линию — это одно и то же.

Проекцию вектора на ось можно рассматривать в двух смыслах: геометрическом и алгебраическом.В геометрическом смысле проекция вектора на ось является вектором, а в алгебраическом смысле — числом. Часто это различие явно не указывается, а скорее понимается из контекста. Мы не будем игнорировать это различие: мы будем использовать термин «», когда речь идет о проекции вектора в геометрическом смысле, и термин «», когда речь идет о проекции вектора в алгебраическом смысле (следующий абзац статьи посвящена численной проекции вектора на ось)…

Теперь перейдем к определению проекции вектора на ось. Для этого не помешает повторить.

Пусть нам даны ось L и ненулевой вектор на плоскости или в трехмерном пространстве. Обозначим проекции точек A и B на прямую L соответственно как A 1 и B 1 и построим вектор. Забегая вперед, предположим, что вектор — это проекция вектора на ось L.

Определение.

Проекция вектора на ось Вектор, начало и конец которого являются проекциями начала и конца данного вектора соответственно.

Проекция вектора на ось L обозначена как.

Чтобы построить векторную проекцию на ось L, нужно опустить перпендикуляры из точек A и B на направленную линию L — основания этих перпендикуляров дадут начало и конец желаемой проекции.

Приведем пример проекции вектора на ось.

Пусть на плоскости введена прямоугольная система координат Oxy и дана некоторая точка.Нарисуем радиус-вектор точки М 1 и построим его проекции на оси координат Ox и Oy. Очевидно, это векторы с координатами и соответственно.

Часто можно услышать о проекции одного вектора на другой ненулевой вектор или о проекции вектора на направление вектора. В данном случае имеется в виду проекция вектора на некоторую ось, направление которой совпадает с направлением вектора (в общем случае существует бесконечно много осей, направления которых совпадают с направлением вектора).Проекция вектора на прямую, направление которой определяет вектор, обозначается как.

Обратите внимание, что если угол между векторами и острый, то векторы и сонаправлены. Если угол между векторами и тупой, то векторы и имеют противоположную направленность. Если вектор равен нулю или перпендикулярен вектору, то проекция вектора на прямую, направление которой определяет вектор, является нулевым вектором.

Числовая проекция вектора на ось — определение, обозначение, примеры нахождения.

Числовая характеристика проекции вектора на ось — это числовая проекция этого вектора на эту ось.

Определение.

Числовая проекция вектора на ось. Число, равное произведению длины этого вектора на косинус угла между этим вектором и вектором, определяющим направление оси.

Числовая проекция вектора на ось L обозначается как (без стрелки вверху), а числовая проекция вектора на ось, определяемую вектором, обозначается как.

В этих обозначениях определение числовой проекции вектора на прямую, направленную как вектор, принимает форму, где — длина вектора, — угол между векторами и.

Итак, у нас есть первые формулы для вычисления числовой проекции вектора :. Эта формула используется, когда известны длина вектора и угол между векторами и. Несомненно, эту формулу можно применить и тогда, когда координаты векторов также известны относительно данной прямоугольной системы координат, но в этом случае удобнее использовать другую формулу, которую мы получим ниже.

Пример.

Вычислить числовую проекцию вектора на линию, направленную как вектор, если длина вектора равна 8, а угол между векторами равен и.

Решение.

Из условия задачи имеем … Осталось только применить формулу, позволяющую определить требуемую числовую проекцию вектора:

Ответ:

Мы знаем, что где — скалярное произведение векторов и. Тогда формула, позволяющая найти числовую проекцию вектора на прямую, направленную как вектор, примет вид… То есть мы можем сформулировать другое определение числовой проекции вектора на ось, которое эквивалентно определению, данному в начале этого параграфа.

Определение.

Числовая проекция вектора на ось , направление которой совпадает с направлением вектора, является отношением скалярного произведения векторов к длине вектора.

Полученную формулу вида удобно использовать для нахождения числовой проекции вектора на прямую, направление которой совпадает с направлением вектора, когда координаты векторов и известны.Покажем это на примерах решения.

Пример.

Известно, что вектор определяет направление оси L. Найдите числовую проекцию вектора на ось L.

Решение.

Формула в виде координат: где и. Используем его, чтобы найти требуемую числовую проекцию вектора на ось L:

Ответ:

Пример.

Два вектора даны относительно прямоугольной системы координат Oxyz в трехмерном пространстве и… Найдите числовую проекцию вектора на ось L, направление которой совпадает с направлением вектора.

Решение.

По координатам векторов и можно вычислить скалярное произведение этих векторов: … Длина вектора по его координатам вычисляется по следующей формуле … Затем формула для определения числовой проекции вектора на L ось в координатах имеет вид.

Применим:

Ответ:

Теперь давайте получим связь между числовой проекцией вектора на ось L, направление которой определяет вектор, и длиной проекции вектора на ось L.Для этого рисуем ось L, откладываем векторы и из точки, лежащей на L, опускаем перпендикуляр с конца вектора на прямую L и строим проекцию вектора на ось L. В зависимости от меры угла между векторами и возможны следующие пять вариантов:

В первом случае очевидно, что значит, то.

Во втором случае в отмеченном прямоугольном треугольнике из определения косинуса угла, следовательно,.

В третьем случае очевидно, что, а, значит, и.

В четвертом случае из определения косинуса угла следует, что, где.

В последнем случае, следовательно,
.

Следующее определение числовой проекции вектора на ось объединяет полученные результаты.

Определение.

Числовая проекция вектора на ось L , направленную как вектор, составляет

Пример.

Длина проекции вектора на ось L, направление которой определяет вектор, составляет. Какова числовая проекция вектора на ось L, если угол между векторами и равен радианам.

Поделитесь статьей с друзьями:

Похожие статьи

Эластичность: напряжение и деформация | Физика

Цели обучения

К концу этого раздела вы сможете:

  • Закон штата Гука.
  • Объясните закон Гука, используя графическое представление между деформацией и приложенной силой.
  • Обсудите три типа деформаций, такие как изменение длины, сдвиг в сторону и изменение объема.
  • Опишите на примерах модуль Юнга, модуль сдвига и объемный модуль.
  • Определите изменение длины с учетом массы, длины и радиуса.

Теперь мы переходим от рассмотрения сил, влияющих на движение объекта (таких как трение и сопротивление), к тем, которые влияют на форму объекта.Если бульдозер втолкнет машину в стену, машина не сдвинется с места, но заметно изменит форму. Изменение формы из-за приложения силы — это деформация . Известно, что даже очень небольшие силы вызывают некоторую деформацию. При малых деформациях наблюдаются две важные характеристики. Во-первых, объект возвращается к своей исходной форме, когда сила снимается, то есть деформация является упругой для небольших деформаций. Во-вторых, размер деформации пропорционален силе, то есть при малых деформациях соблюдается закон Гука.В форме уравнения Закон Гука определяется как

F = k Δ L ,

, где Δ L — величина деформации (например, изменение длины), вызванная силой F , а k — константа пропорциональности, которая зависит от формы и состава объекта, а также направления движения. сила. Обратите внимание, что эта сила является функцией деформации Δ L — она ​​не постоянна, как кинетическая сила трения.Переставляем это на

[латекс] \ displaystyle \ Delta {L} = \ frac {F} {k} [/ latex]

дает понять, что деформация пропорциональна приложенной силе. На рисунке 1 показано соотношение по закону Гука между удлинением Δ L пружины или человеческой кости. Для металлов или пружин область прямой линии, к которой относится закон Гука, намного больше. Кости хрупкие, эластичная область небольшая, а перелом резкий. В конце концов, достаточно большое напряжение материала приведет к его разрушению или разрушению.

Закон Гука

F = kΔL ,

, где Δ L — величина деформации (например, изменение длины), вызванная силой F , а k — константа пропорциональности, которая зависит от формы и состава объекта, а также направления движения. сила.

[латекс] \ displaystyle \ Delta {L} = \ frac {F} {k} [/ latex]

Рис. 1. График зависимости деформации ΔL от приложенной силы F.Прямой отрезок — это линейная область, в которой соблюдается закон Гука. Наклон прямой области [латекс] \ frac {1} {k} [/ latex]. Для больших сил график изогнут, но деформация остается упругой — ΔL вернется к нулю, если сила будет устранена. Еще большие силы деформируют объект до тех пор, пока он окончательно не сломается. Форма кривой возле трещины зависит от нескольких факторов, в том числе от того, как прикладывается сила F . Обратите внимание, что на этом графике наклон увеличивается непосредственно перед трещиной, указывая на то, что небольшое увеличение F дает большое увеличение L рядом с трещиной.

Константа пропорциональности k зависит от ряда факторов материала. Например, гитарная струна из нейлона растягивается при затягивании, и удлинение Δ L пропорционально приложенной силе (по крайней мере, для небольших деформаций). Более толстые нейлоновые струны и струны из стали меньше растягиваются при одной и той же приложенной силе, что означает, что у них больше k (см. Рисунок 2). Наконец, все три струны возвращаются к своей нормальной длине, когда сила снимается, при условии, что деформация мала.Большинство материалов будут вести себя таким образом, если деформация будет меньше примерно 0,1% или примерно 1 часть на 10 3 .

Рис. 2. Одна и та же сила, в данном случае груз (w), приложенная к трем различным гитарным струнам одинаковой длины, вызывает три различных деформации, показанные заштрихованными сегментами. Левая нить из тонкого нейлона, посередине — из более толстого нейлона, а правая — из стали.

Потянитесь немного

Как бы вы измерили константу пропорциональности k резиновой ленты? Если резинка растянулась на 3 см, когда к ней была прикреплена 100-граммовая масса, то насколько она растянулась бы, если бы две одинаковые резинки были прикреплены к одной и той же массе — даже если соединить их параллельно или, наоборот, если связать вместе последовательно?

Теперь мы рассмотрим три конкретных типа деформаций: изменение длины (растяжение и сжатие), сдвиг в сторону (напряжение) и изменения объема.Все деформации считаются небольшими, если не указано иное.

Изменение длины — растяжение и сжатие: модуль упругости

Изменение длины Δ L происходит, когда к проволоке или стержню прилагается сила, параллельная ее длине L 0 , либо растягивая (растяжение), либо сжимая. (См. Рисунок 3.)

Рис. 3. (a) Напряжение. Стержень растягивается на длину ΔL , когда сила прилагается параллельно его длине. (б) Сжатие.Тот же стержень сжимается силами той же величины в противоположном направлении. Для очень малых деформаций и однородных материалов ΔL примерно одинаково для той же величины растяжения или сжатия. При больших деформациях площадь поперечного сечения изменяется при сжатии или растяжении стержня.

Эксперименты показали, что изменение длины (Δ L ) зависит только от нескольких переменных. Как уже отмечалось, Δ L пропорциональна силе F и зависит от вещества, из которого сделан объект.Кроме того, изменение длины пропорционально исходной длине L 0 и обратно пропорционально площади поперечного сечения проволоки или стержня. Например, длинная гитарная струна растягивается больше, чем короткая, а толстая струна растягивается меньше, чем тонкая. Мы можем объединить все эти факторы в одно уравнение для Δ L :

[латекс] \ displaystyle \ Delta {L} = \ frac {1} {Y} \ text {} \ frac {F} {A} L_0 [/ latex],

, где Δ L — изменение длины, F — приложенная сила, Y — коэффициент, называемый модулем упругости или модулем Юнга, который зависит от вещества, A — площадь поперечного сечения, и L 0 — исходная длина.В таблице 1 перечислены значения Y для нескольких материалов. Говорят, что материалы с большим Y имеют большую прочность на разрыв , потому что они меньше деформируются при заданном растяжении или сжатии.

Таблица 1. Модули упругости
Материал Модуль Юнга (растяжение-сжатие) Y (10 9 Н / м 2 ) Модуль сдвига S (10 9 Н / м 2 ) Модуль объемной упругости B (10 9 Н / м 2 )
Алюминий 70 25 75
Кость — напряжение 16 80 8
Кость — компрессия 9
Латунь 90 35 75
Кирпич 15
Бетон 20
Стекло 70 20 30
Гранит 45 20 45
Волосы (человеческие) 10
Твердая древесина 15 10
Чугун литой 100 40 90
Свинец 16 5 50
Мрамор 60 20 70
Нейлон 5
полистирол 3
шелк 6
Паутинка 3
Сталь 210 80 130
Сухожилие 1
ацетон 0.7
Этанол 0,9
Глицерин 4,5
Меркурий 25
Вода 2,2

Модули Юнга не указаны для жидкостей и газов в таблице 1, потому что они не могут быть растянуты или сжаты только в одном направлении. Обратите внимание, что существует предположение, что объект не ускоряется, поэтому на самом деле существуют две приложенные силы величиной F , действующие в противоположных направлениях.Например, струны на Рисунке 3 натягиваются вниз силой величиной х и удерживаются потолком, который также оказывает силу величиной х .

Пример 1. Растяжение длинного кабеля

Подвесные тросы используются для перевозки гондол на горнолыжных курортах. (См. Рис. 4). Рассмотрим подвесной трос, длина которого без опоры составляет 3 км. Рассчитайте степень растяжения стального троса. Предположим, что кабель имеет диаметр 5,6 см и максимальное натяжение, которое он может выдержать, равно 3.0 × 10 6 Н.

Рис. 4. Гондолы перемещаются по подвесным тросам на горнолыжном курорте Гала Юдзава в Японии. (Источник: Руди Херман, Flickr)

Стратегия

Сила равна максимальному натяжению, или F = 3,0 × 10 6 Н. Площадь поперечного сечения π r 2 = 2,46 × 10 –3 м 2 . Уравнение [latex] \ displaystyle \ Delta {L} = \ frac {1} {Y} \ text {} \ frac {F} {A} L_0 [/ latex] можно использовать для определения изменения длины.{2}} \ right) \ left (\ text {3020 m} \ right) \\ & = & \ text {18 m}. \ End {array} [/ latex]

Обсуждение

Это довольно большая длина, но только около 0,6% от длины без опоры. В этих условиях влияние температуры на длину может быть важным.

Кости в целом не ломаются от растяжения или сжатия. Скорее они обычно ломаются из-за бокового удара или изгиба, что приводит к срезанию или разрыву кости. Поведение костей при растяжении и сжатии важно, потому что оно определяет нагрузку, которую кости могут нести.Кости классифицируются как несущие конструкции, такие как колонны в зданиях и деревьях. Несущие конструкции обладают особенностями; колонны в здании имеют стальные арматурные стержни, а деревья и кости — волокнистые. Кости в разных частях тела выполняют разные структурные функции и подвержены разным нагрузкам. Таким образом, кость в верхней части бедренной кости расположена в виде тонких пластин, разделенных костным мозгом, в то время как в других местах кости могут быть цилиндрическими и заполненными костным мозгом или просто твердыми.Люди с избыточным весом имеют тенденцию к повреждению костей из-за длительного сжатия костных суставов и сухожилий.

Другой биологический пример закона Гука встречается в сухожилиях. Функционально сухожилие (ткань, соединяющая мышцу с костью) должно сначала легко растягиваться при приложении силы, но обеспечивать гораздо большую восстанавливающую силу для большего напряжения. На рисунке 5 показана зависимость напряжения от деформации человеческого сухожилия. Некоторые сухожилия имеют высокое содержание коллагена, поэтому деформация или изменение длины относительно невелико; другие, например, опорные сухожилия (например, в ноге), могут изменять длину до 10%.Обратите внимание, что эта кривая напряжения-деформации является нелинейной, поскольку наклон линии изменяется в разных областях. В первой части растяжения, называемой областью пальца, волокна в сухожилии начинают выравниваться в направлении напряжения — это называется разгибание . В линейной области фибриллы будут растянуты, а в области разрушения отдельные волокна начнут разрываться. Простую модель этой взаимосвязи можно проиллюстрировать параллельными пружинами: разные пружины активируются при разной длине растяжения.Примеры этого приведены в задачах в конце этой главы. Связки (ткань, соединяющая кость с костью) ведут себя аналогичным образом.

Рис. 5. Типичная кривая «напряжение-деформация» для сухожилия млекопитающих. Показаны три области: (1) область пальца ноги (2) линейная область и (3) область разрушения.

В отличие от костей и сухожилий, которые должны быть прочными и эластичными, артерии и легкие должны быть легко растяжимыми. Эластичные свойства артерий важны для кровотока. Когда кровь выкачивается из сердца, давление в артериях увеличивается, и стенки артерий растягиваются.Когда аортальный клапан закрывается, давление в артериях падает, и артериальные стенки расслабляются, чтобы поддерживать кровоток. Когда вы чувствуете свой пульс, вы чувствуете именно это — эластичное поведение артерий, когда кровь хлынет через каждый насос сердца. Если бы артерии были жесткими, вы бы не почувствовали пульс. Сердце также является органом с особыми эластичными свойствами. Легкие расширяются за счет мышечного усилия, когда мы вдыхаем, но расслабляемся свободно и эластично, когда мы выдыхаем. Наша кожа особенно эластична, особенно для молодых.Молодой человек может подняться от 100 кг до 60 кг без видимого провисания кожи. С возрастом снижается эластичность всех органов. Постепенное физиологическое старение за счет снижения эластичности начинается в начале 20-х годов.

Пример 2. Расчет деформации: насколько укорачивается нога, когда вы стоите на ней?

Рассчитайте изменение длины кости верхней части ноги (бедренной кости), когда мужчина весом 70,0 кг поддерживает на ней 62,0 кг своей массы, предполагая, что эта кость эквивалентна стержню, равному 40.0 см в длину и 2,00 см в радиусе.

Стратегия

Сила равна поддерживаемому весу, или F = мг = (62,0 кг) (9,80 м / с 2 ) = 607,6 Н, а площадь поперечного сечения равна π r 2 = 1,257 × 10 –3 м 2 . Уравнение [latex] \ displaystyle \ Delta {L} = \ frac {1} {Y} \ text {} \ frac {F} {A} L_0 [/ latex] можно использовать для определения изменения длины.

Решение

Все величины, кроме Δ L , известны.{-5} \ text {m.} \ End {array} [/ latex]

Обсуждение

Это небольшое изменение длины кажется разумным, поскольку, по нашему опыту, кости жесткие. Фактически, даже довольно большие силы, возникающие при напряженных физических нагрузках, не сжимают и не сгибают кости в значительной степени. Хотя кость более жесткая по сравнению с жиром или мышцами, некоторые из веществ, перечисленных в таблице 1, имеют более высокие значения модуля Юнга Y . Другими словами, они более жесткие и обладают большей прочностью на разрыв.

Уравнение изменения длины по традиции перестраивается и записывается в следующем виде:

[латекс] \ displaystyle \ frac {F} {A} = Y \ frac {\ Delta {L}} {L_0} [/ latex].

Отношение силы к площади, [латекс] \ frac {F} {A} [/ latex], определяется как напряжение (измеряется в Н / м 2 ), а отношение изменения длины к длина, [латекс] \ frac {\ Delta {L}} {L_0} [/ latex], определяется как деформация (безразмерная величина). Другими словами, напряжение = Y × деформация.

В этой форме уравнение аналогично закону Гука с напряжением, аналогичным силе, и деформацией, аналогичной деформации. Если снова переписать это уравнение к виду

[латекс] \ displaystyle {F} = YA \ frac {\ Delta {L}} {L_0} [/ latex],

мы видим, что он совпадает с законом Гука с константой пропорциональности

[латекс] \ displaystyle {k} = \ frac {YA} {L_0} [/ latex].

Эта общая идея о том, что сила и вызываемая ею деформация пропорциональны небольшим деформациям, применима к изменениям длины, боковому изгибу и изменениям объема.

Напряжение

Отношение силы к площади, [латекс] \ frac {F} {A} [/ латекс], определяется как напряжение, измеренное в Н / м 2 .

Штамм

Отношение изменения длины к длине, [латекс] \ frac {\ Delta {L}} {L_0} [/ latex], определяется как деформация (безразмерная величина). Другими словами, напряжение = Y × деформация.

Боковое напряжение: Модуль сдвига

На рисунке 6 показано, что подразумевается под боковым напряжением или срезающей силой .Здесь деформация называется Δ x , и она перпендикулярна L 0 , а не параллельна, как при растяжении и сжатии. Деформация сдвига аналогична растяжению и сжатию и может быть описана аналогичными уравнениями. Выражение для деформации сдвига : [латекс] \ displaystyle \ Delta {x} = \ frac {1} {S} \ frac {F} {A} L_0 [/ latex], где S — модуль сдвига ( см. Таблицу 1) и F — сила, приложенная перпендикулярно к L 0 и параллельно площади поперечного сечения A .Опять же, чтобы объект не ускорялся, на самом деле есть две равные и противоположные силы F , приложенные к противоположным граням, как показано на рисунке 6. Уравнение логично — например, легче согнуть длинный тонкий карандаш (маленький A ), чем короткий толстый, и оба гнуть легче, чем аналогичные стальные стержни (большие S ).

Рис. 6. Сила сдвига прилагается перпендикулярно длине L 0 и параллельно области A , создавая деформацию Δx.Вертикальные силы не показаны, но следует иметь в виду, что в дополнение к двум силам сдвига, F , должны существовать поддерживающие силы, чтобы объект не вращался. Искажающие эффекты этих поддерживающих сил игнорируются при этом лечении. Вес объекта также не показан, поскольку он обычно незначителен по сравнению с силами, достаточно большими, чтобы вызвать значительные деформации.

Деформация сдвига

[латекс] \ displaystyle \ Delta {x} = \ frac {1} {S} \ frac {F} {A} L_0 [/ latex],

, где S — модуль сдвига, а F — сила, приложенная перпендикулярно к L 0 и параллельно площади поперечного сечения A .

Изучение модулей сдвига в таблице 1 выявляет некоторые характерные закономерности. Например, для большинства материалов модули сдвига меньше модулей Юнга. Кость — замечательное исключение. Его модуль сдвига не только больше, чем модуль Юнга, но и такой же, как у стали. Это одна из причин того, что кости могут быть длинными и относительно тонкими. Кости могут выдерживать нагрузки, сопоставимые с бетонными и стальными. Большинство переломов костей возникает не из-за сжатия, а из-за чрезмерного скручивания и изгиба.

Позвоночный столб (состоящий из 26 позвоночных сегментов, разделенных дисками) обеспечивает основную опору для головы и верхней части тела. Позвоночник имеет нормальную кривизну для стабильности, но эту кривизну можно увеличить, что приведет к увеличению силы сдвига на нижние позвонки. Диски лучше выдерживают силы сжатия, чем силы сдвига. Поскольку позвоночник не вертикальный, вес верхней части тела влияет на обе части. Беременным женщинам и людям с избыточным весом (с большим животом) необходимо отвести плечи назад, чтобы поддерживать равновесие, тем самым увеличивая искривление позвоночника и тем самым увеличивая сдвигающий компонент напряжения.Увеличенный угол из-за большей кривизны увеличивает поперечные силы вдоль плоскости. Эти более высокие усилия сдвига увеличивают риск травмы спины из-за разрыва дисков. Пояснично-крестцовый диск (клиновидный диск под последними позвонками) особенно подвержен риску из-за своего расположения.

Модули сдвига для бетона и кирпича очень малы; они слишком изменчивы, чтобы их можно было перечислить. Бетон, используемый в зданиях, может выдерживать сжатие, как в колоннах и арках, но очень плохо противостоит сдвигу, который может возникнуть в сильно нагруженных полах или во время землетрясений.Современные конструкции стали возможны благодаря использованию стали и железобетона. Практически по определению жидкости и газы имеют модуль сдвига, близкий к нулю, потому что они текут в ответ на силы сдвига.

Пример 3. Расчет силы, необходимой для деформации: гвоздь не сильно изгибается под нагрузкой

Найдите массу картины, висящей на стальном гвозде, как показано на рисунке 7, учитывая, что гвоздь изгибается только на 1,80 мкм. (Предположим, что модуль сдвига известен с двумя значащими цифрами.)

Рис. 7. Гвоздь, вид сбоку с прикрепленным к нему изображением. Гвоздь очень слабо прогибается (показан намного больше, чем на самом деле) из-за срезающего воздействия поддерживаемого веса. Также показано направленное вверх усилие стенки на гвоздь, иллюстрирующее равные и противоположные силы, приложенные к противоположным поперечным сечениям гвоздя. См. Пример 3 для расчета массы изображения.

Стратегия

Сила F на гвоздь (без учета собственного веса гвоздя) — это вес изображения w .Если мы сможем найти w , то масса изображения будет просто [латекс] \ frac {w} {g} [/ latex]. Уравнение [латекс] \ displaystyle \ Delta {x} = \ frac {1} {S} \ frac {F} {A} L_0 [/ latex] может быть решено для F .

Решение

Решая уравнение [латекс] \ displaystyle \ Delta {x} = \ frac {1} {S} \ frac {F} {A} L_0 [/ latex] для F , мы видим, что все остальные величины могут быть найдены :

[латекс] \ displaystyle {F} = \ frac {SA} {L_0} \ Delta {x} [/ latex]

S находится в таблице 1 и составляет S = 80 × 10 9 Н / м 2 .{-6} \ text {m} \ right) = 51 \ text {N} [/ latex]

Эта сила 51 Н составляет вес w изображения, поэтому масса изображения [латекс] m = \ frac {w} {g} = \ frac {F} {g} = 5.2 \ text {kg} [ /латекс].

Обсуждение

Это довольно массивное изображение, и впечатляет то, что гвоздь прогибается всего на 1,80 мкм — величину, которую невозможно обнаружить невооруженным глазом.

Изменение объема: модуль объемной упругости

Объект будет сжиматься во всех направлениях, если внутренние силы приложены равномерно ко всем его поверхностям, как показано на рисунке 8.Относительно легко сжимать газы и чрезвычайно сложно сжимать жидкости и твердые тела. Например, воздух в винной бутылке сжимается, когда она закупорена. Но если вы попытаетесь закупорить бутылку с полными краями, вы не сможете сжать вино — некоторые из них необходимо удалить, чтобы вставить пробку. Причина такой разной сжимаемости заключается в том, что атомы и молекулы разделены большими пустыми пространствами в газах, но плотно упакованы в жидкостях и твердых телах. Чтобы сжать газ, вы должны сблизить его атомы и молекулы.Чтобы сжать жидкости и твердые тела, вы должны действительно сжать их атомы и молекулы, и очень сильные электромагнитные силы в них препятствуют этому сжатию.

Рис. 8. Внутренняя сила на всех поверхностях сжимает этот куб. Его изменение в объеме пропорционально силе на единицу площади и его первоначальному объему и связано со сжимаемостью вещества.

Мы можем описать сжатие или объемную деформацию объекта уравнением. Во-первых, отметим, что сила, «приложенная равномерно», определяется как имеющая одинаковое напряжение или отношение силы к площади [латекс] \ frac {F} {A} [/ латекс] на всех поверхностях.Произведенная деформация представляет собой изменение объема Δ V , которое, как было обнаружено, ведет себя очень похоже на сдвиг, растяжение и сжатие, описанные ранее. (Это неудивительно, поскольку сжатие всего объекта эквивалентно сжатию каждого из его трех измерений.) Связь изменения объема с другими физическими величинами определяется выражением [latex] \ displaystyle \ Delta {V} = \ frac {1} {B} \ frac {F} {A} V_0 [/ latex], где B — модуль объемной упругости (см. Таблицу 1), V 0 — исходный объем, а [латекс] \ frac {F} {A} [/ latex] — это сила на единицу площади, равномерно приложенная внутрь ко всем поверхностям.Обратите внимание, что объемные модули для газов не приводятся.

Какие есть примеры объемного сжатия твердых тел и жидкостей? Одним из практических примеров является производство алмазов промышленного качества путем сжатия углерода с чрезвычайно большой силой на единицу площади. Атомы углерода перестраивают свою кристаллическую структуру в более плотно упакованный узор алмазов. В природе аналогичный процесс происходит глубоко под землей, где чрезвычайно большие силы возникают из-за веса вышележащего материала. Еще один естественный источник больших сжимающих сил — давление, создаваемое весом воды, особенно в глубоких частях океанов.Вода оказывает внутреннее воздействие на все поверхности погружаемого объекта и даже на саму воду. На больших глубинах вода ощутимо сжата, как показано в следующем примере.

Пример 4. Расчет изменения объема с деформацией: насколько вода сжимается на глубинах Великого океана?

Рассчитайте частичное уменьшение объема [латекс] \ left (\ frac {\ Delta {V}} {V_0} \ right) [/ latex] для морской воды на глубине 5,00 км, где сила на единицу площади составляет 5,00 × 10 7 Н / м 2 .

Стратегия

Уравнение [латекс] \ displaystyle \ Delta {V} = \ frac {1} {B} \ frac {F} {A} V_0 [/ latex] является правильным физическим соотношением. Все величины в уравнении, кроме [latex] \ frac {\ Delta {V}} {V_0} [/ latex], известны.

Решение

Решение неизвестного [латекса] \ frac {\ Delta {V}} {V_0} [/ latex] дает [latex] \ displaystyle \ frac {\ Delta {V}} {V_0} = \ frac {1} {B } \ frac {F} {A} [/ латекс].

Замена известных значений значением модуля объемной упругости B из таблицы 1,

[латекс] \ begin {array} {lll} \ frac {\ Delta {V}} {V_0} & = & \ frac {5.2} \\ & = & 0.023 = 2.3 \% \ end {array} [/ latex]

Обсуждение

Хотя это и поддается измерению, это незначительное уменьшение объема, учитывая, что сила на единицу площади составляет около 500 атмосфер (1 миллион фунтов на квадратный фут). Жидкости и твердые вещества чрезвычайно трудно сжимать.

И наоборот, очень большие силы создаются жидкостями и твердыми телами, когда они пытаются расшириться, но им это мешает, что эквивалентно их сжатию до меньшего, чем их нормальный объем.Это часто происходит, когда содержащийся в нем материал нагревается, поскольку большинство материалов расширяются при повышении их температуры. Если материалы сильно стеснены, они деформируют или ломают свой контейнер. Другой очень распространенный пример — замерзание воды. Вода, в отличие от большинства материалов, расширяется при замерзании, и она может легко сломать валун, разорвать биологическую клетку или сломать блок двигателя, который встанет у нее на пути.

Другие типы деформаций, такие как кручение или скручивание, ведут себя аналогично рассмотренным здесь деформациям растяжения, сдвига и объемной деформации.

Сводка раздела

  • Закон Гука определяется выражением [латекс] F = k \ Delta {L} [/ latex], где [latex] \ Delta {L} [/ latex] — величина деформации (изменение длины), F — это приложенная сила, а k — константа пропорциональности, которая зависит от формы и состава объекта, а также от направления силы. Связь между деформацией и приложенной силой также может быть записана как [latex] \ displaystyle \ Delta L = \ frac {1} {Y} \ frac {F} {A} {L} _ {0} [/ latex] , где Y — это модуль Юнга , , который зависит от вещества, A — площадь поперечного сечения, а [латекс] {L} _ {0} [/ latex] — исходная длина.
  • Отношение силы к площади, [латекс] \ frac {F} {A} [/ латекс], определяется как напряжение , измеренное в Н / м 2 .
  • Отношение изменения длины к длине, [латекс] \ frac {\ Delta L} {{L} _ {0}} [/ latex], определяется как деформация (безразмерная величина). Другими словами, [латекс] \ текст {напряжение} = Y \ times \ text {напряжение} [/ латекс].
  • Выражение деформации сдвига [латекс] \ displaystyle \ Delta x = \ frac {1} {S} \ frac {F} {A} {L} _ {0} [/ latex], где S — модуль сдвига и F — это сила, приложенная перпендикулярно [латексу] {L} _ {\ text {0}} [/ latex] и параллельно площади поперечного сечения A .
  • Связь изменения объема с другими физическими величинами определяется выражением [latex] \ displaystyle \ Delta V = \ frac {1} {B} \ frac {F} {A} {V} _ {0} [/ latex ], где B — объемный модуль, [latex] {V} _ {\ text {0}} [/ latex] — исходный объем, а [latex] \ frac {F} {A} [/ latex] — сила на единицу площади, равномерно приложенная внутрь ко всем поверхностям.

Концептуальные вопросы

  1. Эластичные свойства артерий важны для кровотока. Объясните важность этого с точки зрения характеристик кровотока (пульсирующий или непрерывный).
  2. Что вы чувствуете, когда щупаете пульс? Измерьте частоту пульса в течение 10 секунд и 1 минуты. Есть ли разница в 6 раз?
  3. Изучите различные типы обуви, включая спортивную обувь и шлепанцы. С точки зрения физики, почему нижние поверхности устроены именно так? Какие различия будут иметь для этих поверхностей сухие и влажные условия?
  4. Ожидаете ли вы, что ваш рост будет отличаться в зависимости от времени суток? Почему или почему нет?
  5. Почему белка может спрыгнуть с ветки дерева на землю и убежать целой, а человек может сломать кость при таком падении?
  6. Объясните, почему беременные женщины часто страдают растяжением спины на поздних сроках беременности.
  7. Уловка старого плотника, чтобы не допустить сгибания гвоздей при забивании их в твердый материал, заключается в том, чтобы крепко удерживать центр гвоздя плоскогубцами. Почему это помогает?
  8. Когда стеклянная бутылка, полная уксуса, нагревается, и уксус, и стекло расширяются, но уксус расширяется значительно больше с температурой, чем стекло. Бутылка разобьется, если наполнить ее до плотно закрытой крышки. Объясните, почему, а также объясните, как воздушный карман над уксусом предотвратит разрыв.(Это функция воздуха над жидкостями в стеклянных контейнерах.)

Задачи и упражнения

  1. Во время циркового представления один артист качается вверх ногами, висит на трапеции, держа другого, также перевернутого, за ноги. Если восходящая сила, действующая на более низкую спортсменку, в три раза превышает ее вес, насколько растягиваются кости (бедра) в ее верхних конечностях? Вы можете предположить, что каждый из них эквивалентен одинаковому стержню длиной 35,0 см и радиусом 1,80 см. Ее масса 60.0 кг.
  2. Во время схватки борец 150 кг ненадолго встает на одну руку во время маневра, призванного сбить с толку его и без того умирающего противника. Насколько укорачивается длина кости плеча? Кость может быть представлена ​​однородным стержнем длиной 38,0 см и радиусом 2,10 см.
  3. (a) «Грифель» в карандашах представляет собой состав графита с модулем Юнга примерно 1 × 10 9 Н / м 2 . Вычислите изменение длины грифеля в автоматическом карандаше, если постучите им прямо по карандашу с силой 4.0 Н. Шнур диаметром 0,50 мм и длиной 60 мм. б) разумен ли ответ? То есть согласуется ли это с тем, что вы наблюдали при использовании карандашей?
  4. антенн телевещания — самые высокие искусственные сооружения на Земле. В 1987 году физик весом 72,0 кг разместил себя и 400 кг оборудования на вершине одной антенны высотой 610 м для проведения гравитационных экспериментов. Насколько была сжата антенна, если считать ее эквивалентом стального цилиндра радиусом 0,150 м?
  5. (a) На сколько стоит 65.Альпинист весом 0 кг натягивает нейлоновую веревку диаметром 0,800 см, когда она висит на 35,0 м ниже скалы? б) Соответствует ли ответ тому, что вы наблюдали для нейлоновых веревок? Имел бы смысл, если бы веревка была на самом деле эластичным шнуром?
  6. Полый алюминиевый флагшток высотой 20,0 м по жесткости эквивалентен твердому цилиндру диаметром 4,00 см. Сильный ветер изгибает полюс так же, как горизонтальная сила в 900 Н. Насколько далеко в сторону прогибается вершина шеста?
  7. По мере бурения нефтяной скважины каждая новая секция бурильной трубы выдерживает собственный вес, а также вес трубы и бурового долота под ней.Рассчитайте растяжение новой стальной трубы длиной 6,00 м, которая поддерживает 3,00 км трубы, имеющей массу 20,0 кг / м, и буровое долото 100 кг. Труба эквивалентна по жесткости сплошному цилиндру диаметром 5 см.
  8. Вычислите усилие, которое настройщик рояля применяет для растяжения стальной рояльной струны на 8,00 мм, если проволока изначально имеет диаметр 0,850 мм и длину 1,35 м.
  9. Позвонок подвергается действию силы сдвига 500 Н. Найдите деформацию сдвига, принимая позвонок в виде цилиндра 3.00 см в высоту и 4,00 см в диаметре.
  10. Диск между позвонками в позвоночнике подвергается действию силы сдвига 600 Н. Найдите его деформацию сдвига, принимая модуль сдвига 1 × 10 9 Н / м 2 . Диск эквивалентен сплошному цилиндру высотой 0,700 см и диаметром 4,00 см.
  11. При использовании ластика для карандашей вы прикладываете вертикальную силу 6,00 Н на расстоянии 2,00 см от соединения ластика с твердой древесиной. Карандаш имеет диаметр 6,00 мм и держится под углом 20 °.0º к горизонтали. а) Насколько дерево прогибается перпендикулярно своей длине? б) Насколько он сжат в продольном направлении?
  12. Чтобы рассмотреть влияние проводов, подвешенных на столбах, мы возьмем данные из рисунка 9, на котором были рассчитаны натяжения проводов, поддерживающих светофор. Левая проволока образовывала угол 30,0 ° ниже горизонтали с вершиной своего столба и выдерживала натяжение 108 Н. Полый алюминиевый столб высотой 12,0 м по жесткости эквивалентен твердому цилиндру диаметром 4,50 см.а) Насколько он наклонен в сторону? б) Насколько он сжат?

    Рисунок 9. Светофор подвешен на двух тросах. (б) Некоторые из задействованных сил. (c) Здесь показаны только силы, действующие на систему. Также показана схема свободного движения светофора. (d) Силы, проецируемые на вертикальную ( x ) и горизонтальную ( x ) оси. Горизонтальные составляющие натяжения должны компенсироваться, а сумма вертикальных составляющих натяжений должна равняться весу светофора.{-2} [/ латекс]). Какую силу на единицу площади вода может оказывать на емкость при замерзании? (В этой задаче допустимо использовать объемный модуль упругости воды.) (B) Удивительно ли, что такие силы могут разрушать блоки двигателя, валуны и тому подобное?

  13. Эта проблема возвращается к канатоходцу, изображенному на Рисунке 10, который создал натяжение 3,94 × 10 3 Н в канате, образующем угол 5,0 ° ниже горизонтали с каждой опорной стойкой. Подсчитайте, насколько это натяжение растягивает стальную проволоку, если она изначально была длиной 15 м и равной 0.50 см в диаметре.

    Рис. 10. Вес канатоходца вызывает провисание каната на 5,0 градуса. Интересующая здесь система — это точка на проволоке, на которой стоит канатоходец.

  14. Полюс на Рисунке 11 находится под изгибом 90,0º в линии электропередачи и поэтому подвергается большей силе сдвига, чем полюса на прямых участках линии. Натяжение в каждой линии составляет 4,00 × 10 4 Н при показанных углах. Шест 15,0 м в высоту, 18,0 см в диаметре и, как считается, имеет вдвое меньшую жесткость, чем древесина твердых пород.(а) Рассчитайте сжатие полюса. (б) Найдите, насколько он изгибается и в каком направлении. (c) Найдите натяжение в растяжке, используемой для удержания вехи прямо, если она прикреплена к верхней части столба под углом 30,0 ° к вертикали. (Ясно, что растяжка должна быть в направлении, противоположном изгибу.)

Рис. 11. Этот телефонный столб находится под углом 90 ° к линии электропередачи. Оттяжка прикрепляется к вершине мачты под углом 30º к вертикали.

Глоссарий

Сила сопротивления: F D , как оказалось, пропорционально квадрату скорости объекта; математически

[латекс] \ begin {array} \\ F _ {\ text {D}} \ propto {v} ^ 2 \\ F _ {\ text {D}} = \ frac {1} {2} C \ rho {Av } ^ 2 \ end {array} [/ latex],

, где C — коэффициент лобового сопротивления, A — площадь объекта, обращенного к жидкости, а ρ — плотность жидкости.

Закон Стокса: F s = 6 πrη v , где r — радиус объекта, η — вязкость жидкости, а v — величина объекта. скорость.

Решения проблем и упражнения

1. 1.90 × 10 −3 см

3. (а) 1 мм; (б) Это кажется разумным, поскольку кажется, что поводок немного сжимается, когда вы на него нажимаете.

5. (а) 9 см; (б) Это кажется разумным для нейлоновой веревки для лазания, поскольку она не должна сильно растягиваться.

7. 8,59 мм

9. 1.49 × 10 −7 м

11. (а) 3.99 × 10 −7 м; (б) 9,67 × 10 −8 м

13. 4 × 10 6 Н / м 2 . Это примерно 36 атм, больше, чем может выдержать обычная банка.

15. 1,4 см


Обзор

векторов и снарядов — с ответами №2

Перейдите к:

Обзор сессии Главная — Список тем

Векторы и снаряды — Главная || Версия для печати || Вопросы и ссылки

Ответы на вопросы: Все || # 1-9 || # 10-45 || # 46-55 || # 56-72


[# 10 | # 11 | # 12 | # 13 | # 14 | # 15 | # 16 | # 17 | # 18 | # 19 | # 20 | # 21 | # 22 | # 23 | # 24 | # 25 | # 26 | # 27 | # 28 | # 29 | # 30 | # 31 | # 32 | # 33 | # 34 | # 35 | # 36 | # 37 | # 38 | # 39 | # 40 | # 41 | # 42 | # 43 | # 44 | # 45]

Часть B: множественный выбор

10.Если сложить два вектора смещения 6 метров и 8 метров (с разными направлениями), то результат может находиться в диапазоне от ___ метров до ___ метров.

а. 0, 48

г. 1,33, 48

г. 0, 14

г. 2, 14

e. …ерунда! Такого прогноза сделать нельзя.

ф. … ерунда! Можно сделать прогноз, но ни один из этих вариантов не верен.

Ответ: D

Векторная сумма 6,0 метра и 8,0 метра будет наибольшей, если их сложить в одном направлении; что даст в результате 14 метров. Векторная сумма 6,0 метра и 8,0 метра будет наименьшей, если их сложить в противоположном направлении; что даст результат 2.0 метров. Анимация, изображающая различные результирующие 6,0 метра, добавленные к 8,0 метрам в различных направлениях, показана на отдельной странице. Смотрите анимацию.

[# 10 | # 11 | # 12 | # 13 | # 14 | # 15 | # 16 | # 17 | # 18 | # 19 | # 20 | # 21 | # 22 | # 23 | # 24 | # 25 | # 26 | # 27 | # 28 | # 29 | # 30 | # 31 | # 32 | # 33 | # 34 | # 35 | # 36 | # 37 | # 38 | # 39 | # 40 | # 41 | # 42 | # 43 | # 44 | # 45]


11. Три вектора добавляются по правилам сложения векторов. Четвертый вектор рисуется от хвоста первого вектора к голове последнего вектора.Этот четвертый вектор обозначается как ____.

а. равновесие

г. гипотенуза

г. в результате

г. ошибка

Ответ: C

Результирующий результат представляет собой результат сложения двух или более векторов. На диаграмме сложения масштабированных векторов результат всегда тянется от хвоста первого вектора к голове последнего вектора.

[# 10 | # 11 | # 12 | # 13 | # 14 | # 15 | # 16 | # 17 | # 18 | # 19 | # 20 | # 21 | # 22 | # 23 | # 24 | # 25 | # 26 | # 27 | # 28 | # 29 | # 30 | # 31 | # 32 | # 33 | # 34 | # 35 | # 36 | # 37 | # 38 | # 39 | # 40 | # 41 | # 42 | # 43 | # 44 | # 45]


12. ВЕРНО или НЕВЕРНО:

Порядок добавления векторов повлияет на конечный результат.

Ответ: B

Порядок, в котором добавляются векторы, не влияет на величину или направление результирующего.A + B + C дает тот же результат, что и B + C + A, и тот же результат, что и A + C + B. Анимация, изображающая сложение пяти векторов в трех разных порядках, отображается на отдельной странице. . Смотрите анимацию.

[# 10 | # 11 | # 12 | # 13 | # 14 | # 15 | # 16 | # 17 | # 18 | # 19 | # 20 | # 21 | # 22 | # 23 | # 24 | # 25 | # 26 | # 27 | # 28 | # 29 | # 30 | # 31 | # 32 | # 33 | # 34 | # 35 | # 36 | # 37 | # 38 | # 39 | # 40 | # 41 | # 42 | # 43 | # 44 | # 45]


13. Вектор A направлен на север, а вектор B — на восток.Какая из следующих диаграмм сложения векторов лучше всего представляет сложение векторов A и B и последующий результат?


Ответ: E

Если используется метод «голова к хвосту» (иногда называемый методом треугольника), то хвост B должен быть нарисован, начиная с головы A. Обе диаграммы D и E показывают это. Затем полученный результат следует переместить от хвоста A к голове B (что не показано на диаграмме D). Также существует метод параллелограмма для сложения векторов.В этом методе хвосты двух векторов помещаются вместе (как на диаграммах A и B). Затем следует нарисовать параллелограмм с двумя векторами, образующими смежные стороны параллелограмма. Результирующая проводится от хвостов векторов к противоположному углу параллелограмма. Это сделано неправильно на диаграммах A и B. Диаграммы C и F не похожи ни на один (точный) метод сложения векторов, известный человечеству.

[# 10 | # 11 | # 12 | # 13 | # 14 | # 15 | # 16 | # 17 | # 18 | # 19 | # 20 | # 21 | # 22 | # 23 | # 24 | # 25 | # 26 | # 27 | # 28 | # 29 | # 30 | # 31 | # 32 | # 33 | # 34 | # 35 | # 36 | # 37 | # 38 | # 39 | # 40 | # 41 | # 42 | # 43 | # 44 | # 45]


14.При добавлении вектора B к вектору A геометрически (или графически) с использованием метода «голова к хвосту» результат отрисовывается от ____ до ____.

а. голова А, хвост В

г. хвост А, голова Б

г. голова B, хвост A

г. хвост Б, голова А

Ответ: B

Добавление вектора B к вектору A эквивалентно слову A + B.В таком случае сначала рисуется A, затем B рисуется с хвостом во главе A. Наконец, результат вычерчивается от хвоста первого вектора (A) до головы последнего вектора ( Б) .

[# 10 | # 11 | # 12 | # 13 | # 14 | # 15 | # 16 | # 17 | # 18 | # 19 | # 20 | # 21 | # 22 | # 23 | # 24 | # 25 | # 26 | # 27 | # 28 | # 29 | # 30 | # 31 | # 32 | # 33 | # 34 | # 35 | # 36 | # 37 | # 38 | # 39 | # 40 | # 41 | # 42 | # 43 | # 44 | # 45]

Используйте следующие векторные диаграммы сложения для вопросов # 15- # 20.


15. Какое из следующих уравнений сложения векторов показано на диаграмме 1?

а. А + В = С

г. А + С = В

г. В + С = А

г. В + А = С

e. С + В = А

ф.С + А = В

г. Ни один из этих


16. Какое из следующих уравнений сложения векторов показано на диаграмме 2?

а. А + В = С

г. А + С = В

г. В + С = А

г.В + А = С

e. С + В = А

ф. С + А = В

г. Ни один из этих


17. Какое из следующих уравнений сложения векторов показано на диаграмме 3?

а. А + В = С

г.А + С = В

г. В + С = А

г. В + А = С

e. С + В = А

ф. С + А = В

г. Ни один из этих


18. Какое из следующих уравнений сложения векторов показано на диаграмме 4?

а.А + В = С

г. А + С = В

г. В + С = А

г. В + А = С

e. С + В = А

ф. С + А = В

г. Ни один из этих

Ответы на вопросы №15- №18:

15.В

16. А

17. E

18. G

Векторов добавляются методом «голова к хвосту», и результат протягивается от хвоста первого вектора к началу последнего вектора. Таким образом, если добавляются два вектора — скажем, B добавляется к A (как в A + B) — тогда сначала рисуется A, а хвост B помещается в начало A. Получившийся результат рисуется так, чтобы его хвост находился в хвосте вектора. A и его голова в начале B. Таким образом, когда хвосты двух векторов видны связанными, один из векторов является результирующим, а другой — первым добавляемым вектором.И когда головы двух векторов видны соединенными, один из векторов является результирующим, а другой — вторым добавляемым вектором. Эти принципы могут быть применены для ответа на поставленные выше вопросы.

Обратите внимание, что в вопросе 18 нет ни одного вектора, который проводится от хвоста одного вектора к началу другого вектора. Таким образом, результат не выводится. Можно сказать, что диаграмма показывает, что A + B + C = 0.

19. Рассмотрите величину и направление векторов A и B, как показано на Диаграмме 1 выше.Какая из следующих диаграмм представит B — A = R?

Ответ: B

Вычитание A из B эквивалентно добавлению отрицательного числа A к B. То есть B — A = B + (-A). Негатив вектора — это просто тот же вектор, нарисованный в противоположном направлении. Правильным ответом должна быть диаграмма, на которой сначала изображается буква B. Затем во главе B должен быть нарисован хвост вектора, указывающий в направлении, противоположном направлению A. Это показано на обеих диаграммах B и C.Затем полученный результат должен быть проведен от хвоста B к голове -A. Это не показано на диаграмме C.


20. Рассмотрите величину и направление векторов B и C, как показано на Диаграмме 2 выше. Какая из следующих диаграмм представит C — B = R?

Ответ: C

Вычитание B из C эквивалентно добавлению отрицательного числа B к C. То есть C — B = C + (-B). Негатив вектора — это просто тот же вектор, нарисованный в противоположном направлении.Правильным ответом должна быть диаграмма, на которой сначала нарисован C. Затем во главе C следует нарисовать хвост вектора, указывающий в направлении, противоположном направлению B. Это показано на обеих диаграммах B и C. Полученный результат затем должен быть проведен от хвоста C к голове -B. Это не показано на диаграмме B.

[# 10 | # 11 | # 12 | # 13 | # 14 | # 15 | # 16 | # 17 | # 18 | # 19 | # 20 | # 21 | # 22 | # 23 | # 24 | # 25 | # 26 | # 27 | # 28 | # 29 | # 30 | # 31 | # 32 | # 33 | # 34 | # 35 | # 36 | # 37 | # 38 | # 39 | # 40 | # 41 | # 42 | # 43 | # 44 | # 45]

21.Векторная сумма (только звездная величина) 25,0 м, север + 18,0 м, восток ___ м.

а. 7,00

г. 21,5

г. 30,8

г. 35,8

e. 43,0

ф. 54,2

г.949

ч. Ни один из этих

Ответ: C

Эти два вектора направлены под прямым углом друг к другу. В таких случаях векторная сумма может быть определена с помощью теоремы Пифагора. Результирующее (R) равно квадратному корню из суммы квадратов двух добавляемых векторов. То есть R = Sqrt (A 2 + B 2 ), где A и B — два вектора, которые складываются вместе.Таким образом,

R = Площадь [(25,0 м) 2 + (18,0 м) 2 ] = Площадь [(625 м 2 ) + (324 м 2 )] = Площадь (949 м 2 )

R = 30,805 м = ~ 30,8 м

[# 10 | # 11 | # 12 | # 13 | # 14 | # 15 | # 16 | # 17 | # 18 | # 19 | # 20 | # 21 | # 22 | # 23 | # 24 | # 25 | # 26 | # 27 | # 28 | # 29 | # 30 | # 31 | # 32 | # 33 | # 34 | # 35 | # 36 | # 37 | # 38 | # 39 | # 40 | # 41 | # 42 | # 43 | # 44 | # 45]

22. Векторная сумма (только звездная величина) 32,0 м, север + 41.0 м, запад ___ м.

а. 9.00

г. 36,5

г. 38,0

г. 52,0

e. 73,0

ф. 128

г. 2,70 х 10 3

ч.Ни один из этих

Ответ: D

Как и в вопросе № 21 выше, эти два вектора направлены под прямым углом друг к другу. Теорема Пифагора может использоваться, чтобы определить результат их суммы. Результирующее (R) равно квадратному корню из суммы квадратов двух добавляемых векторов.

R = Sqrt (A 2 + B 2 )

R = Sqrt [(32,0 м) 2 + (41,0 м) 2 ] = Sqrt [(1024 м 2 ) + (1681 м 2 )] = Sqrt (2705 м 2 )

R = 52.010 м = ~ 52,0 м

[# 10 | # 11 | # 12 | # 13 | # 14 | # 15 | # 16 | # 17 | # 18 | # 19 | # 20 | # 21 | # 22 | # 23 | # 24 | # 25 | # 26 | # 27 | # 28 | # 29 | # 30 | # 31 | # 32 | # 33 | # 34 | # 35 | # 36 | # 37 | # 38 | # 39 | # 40 | # 41 | # 42 | # 43 | # 44 | # 45]


Используйте схему ниже, чтобы ответить на вопросы # 23- # 28. Каждый квадрат на диаграмме представляет собой площадь 20 х 20 метров.


23. Если человек идет от D к H, от G к C, то пройденное расстояние составляет ____ метров.

а. 128

г. 180

г. 401

г. 460

e. 480

ф. 533

г. 620

ч. Ни один из этих

Ответ: F

Расстояние — это скалярная величина, и сложить три скаляра так же просто, как арифметически сложить три числа.Однако, поскольку первый из трех этапов этого путешествия не проходит через восток, запад, север или юг, для определения расстояния для этого этапа требуется использование теоремы Пифагора. То есть ходьба от D к H эквивалентна переходу на 140 метров на восток (7 квадратов) и 160 метров (8 квадратов) на юг. Таким образом, расстояние от D до H составляет 212,6 метра — Sqrt [(140 м) 2 + (160 м) 2 ]. Расстояние от H до G составляет 80 метров (4 квадрата), а от G до C — 240 метров (12 квадратов).Общее пройденное расстояние составляет 212,6 м + 80 м + 240 м = 532,6 м = ~ 533 м .

24. Если человек идет от D к H, от G к C, то величина смещения составляет ____ метров.

а. 128

г. 180

г. 401

г. 460

e.480

ф. 533

г. 620

ч. Ни один из этих

Ответ: A

Смещение — это векторная величина, которая показывает, как далеко от нашего места находится объект в конце движения относительно начала движения. Смещение не зависит от пути, пройденного от начальной позиции до конечной позиции, а только от расстояния вектора, нарисованного от начала до конца.Результирующая для трех сегментов этого блуждания просто переносится из D в C. Расстояние этого результирующего результата находится с помощью теоремы Пифагора. Результирующий простирается на юг на 80 метров (4 квадрата) и на запад на 100 метров (5 квадратов). Результирующая величина имеет величину Sqrt [(80 м) 2 + (100 м) 2 ] или 128 метров.


25. Если человек идет от D к H, от G к C, то направление смещения составляет ___ градусов (при измерении против часовой стрелки с востока).

а. 39

г. 51

г. 53

г. 217

e. 219

ф. 231

г. 233

ч. Ни один из этих

Ответ: E

Как обсуждалось в предыдущей задаче, результирующая область простирается на юг на 80 метров (4 квадрата) и на запад на 100 метров (5 квадратов).Это показано на диаграмме справа. Эти две части результирующего вектора могут быть объединены с тригонометрической функцией для определения угла тета. Угол тета составляет примерно 38,7 градуса. Это не направление результирующего смещения, а просто угол между вектором смещения и западным направлением. Условие, используемое для выражения направления вектора, заключается в измерении угла поворота против часовой стрелки с востока. Итак, в данном случае направление 180 градусов + 38.7 градусов или ~ 219 градусов.


26. Если человек идет от H до E, от C до G, то пройденное расстояние составляет ____ метров.

а. 80.

г. 240.

г. 253

г. 333

e. 493

ф.560.

г. 640.

ч. Ни один из этих

Ответ: E

Расстояние — это скалярная величина, и сложить три скаляра так же просто, как сложить отдельные расстояния трех ног так же просто, как сложить расстояния. арифметически. Задача упрощается тем, что первые два отрезка пути проходят по одному и тому же отрезку линии.Остановку в точке E можно не принимать во внимание, поскольку это просто точка на линии от точки H до точки C. То есть HE + EC = HC . Поскольку линейный сегмент HC не простирается на восток, запад, север или юг, определение расстояния для этого отрезка требует использования теоремы Пифагора. То есть ходьба от H к C эквивалентна ходу на 240 метров на запад (12 квадратов) и 80 метров (4 квадрата) на север. Таким образом, расстояние от D до H составляет 253,0 метра — Sqrt [(240 м) 2 + (80 м) 2 ].Расстояние от H до E до C составляет 253 метра, а от C до G — 240 метров (12 квадратов). Общее пройденное расстояние составляет 253 м + 240 м = ~ 493 м .

27. Если человек идет от H к E, от C к G, то величина смещения составляет ____ метров.

а. 80.

г. 240.

г. 253

г.333

e. 493

ф. 560.

г. 640.

ч. Ни один из этих

Ответ: A

Смещение — это векторная величина, которая показывает, как далеко от нашего места находится объект в конце движения относительно начала движения.Смещение не зависит от пути, пройденного от начальной позиции до конечной позиции, а только от расстояния вектора, нарисованного от начала до конца. Результирующая величина для трех сегментов этой прогулки просто вычерчивается от H до G. Расстояние этой результирующей составляющей просто 80 метров (4 квадрата) на север.


28. Если человек идет от H к E, от C к G, то направление смещения составляет ___ градусов (при измерении против часовой стрелки с востока).

а.0

г. 18.

г. 72

г. 90.

e. 108

ф. 162

г. 342

ч. Ни один из этих

Ответ: D

Конечный пункт назначения G находится к северу от исходного местоположения.Таким образом, смещение направлено на север (на 90 градусов).

[# 10 | # 11 | # 12 | # 13 | # 14 | # 15 | # 16 | # 17 | # 18 | # 19 | # 20 | # 21 | # 22 | # 23 | # 24 | # 25 | # 26 | # 27 | # 28 | # 29 | # 30 | # 31 | # 32 | # 33 | # 34 | # 35 | # 36 | # 37 | # 38 | # 39 | # 40 | # 41 | # 42 | # 43 | # 44 | # 45]

Используйте следующую схему для вопросов # 29- # 33. На схеме показано речное судно, начинающееся в позиции А на восточном берегу реки. Лодка направляется к позиции B (точка прямо через реку от точки A) со скоростью 3.8 м / с. Но из-за течения со скоростью 1,8 м / с лодка приземляется на западном берегу реки в точке C, расположенной ниже по течению от B. Ширина реки (d через ) составляет 86,4 метра.

[# 10 | # 11 | # 12 | # 13 | # 14 | # 15 | # 16 | # 17 | # 18 | # 19 | # 20 | # 21 | # 22 | # 23 | # 24 | # 25 | # 26 | # 27 | # 28 | # 29 | # 30 | # 31 | # 32 | # 33 | # 34 | # 35 | # 36 | # 37 | # 38 | # 39 | # 40 | # 41 | # 42 | # 43 | # 44 | # 45]


29. Величина результирующей скорости лодки ____ м / с.

а. 1,8

г. 2,0

г. 3,4

г. 3,8

e. 4,2

ф. 5,6

г.11

ч. Ни один из этих

Ответ: E

Это случай, когда лодка пересекает реку с помощью своего двигателя.Мотор позволяет лодке преодолевать 3,8 метра в сторону противоположного берега каждую секунду. Река течет на юг, каждую секунду унося лодку вниз на 1,8 метра. Результирующая скорость — это просто векторная сумма этих двух отдельных скоростей. Поскольку эти две скорости расположены под прямым углом друг к другу, векторная сумма может быть определена с помощью теоремы Пифагора.

v результат 2 = (3,8 м / с) 2 + (1,8 м / с) 2

против , результат = Sqrt [(3.8 м / с) 2 + (1,8 м / с) 2 ]

v результат = 4,2 м / с

30. Направление результирующей скорости лодки ____ м / с.

а. 0

г. 18.

г. 712

г. 90.

e.108

ф. 162

г. 342

ч. Ни один из этих

Ответ: H

Направление результирующей скорости — юго-восток. Это помещает вектор в третий квадрант с направлением где-то между 180 и 270 градусами. Точный угол можно определить, если угол тета определяется с помощью тригонометрии.Тета — это угол, который результирующая скорость (красный вектор на диаграмме справа) составляет относительно западного направления. Этот угол можно найти с помощью функции касательной. Работа показана справа.

Угол тета составляет 25,3 градуса. Фактическое направление, измеренное как угол поворота против часовой стрелки с учетом восточного направления, составляет 180 градусов плюс 25,3 градуса. Это будет 205,3 градуса .

31. Время, необходимое лодке для пересечения 86.Река шириной 4 м занимает ___ секунд.

а. 4,2

г. 15

г. 21

г. 23

e. 48

ф. Ни один из этих

Ответ: D

Мотор позволяет лодке двигаться 3.8 метров в сторону противоположного берега каждую секунду. Длина пути от берега до берега составляет 86,4 метра. (Присутствие течения не влияет на ширину или расстояние от берега до берега.) Время перехода через реку можно рассчитать, исходя из ширины реки и скорости лодки, используя уравнение v = d / t. Преобразуя уравнение для решения относительно t, получаем

т = д / v

t = (86,4 м) / (3,8 м / с) = ~ 23 с (без закругления 22,7 с)


32.Местоположение C — это место, где лодка в конечном итоге приземляется на противоположном берегу. Каково расстояние от точки B до точки C.

а. 37

г. 41

г. 78

г. 86

e. 96

ф.180

г. 2,0 х 10 2

ч. Ни один из этих

Ответ: B

Расстояние, на котором точка C находится ниже по течению от точки B, математически связано со скоростью реки и временем перехода через реку. Расстояние можно рассчитать, умножив текущую скорость на время перехода через реку.

d ниже по течению = v р. • t

d ниже по потоку = (1.8 м / с) • (22,7 с)

d ниже по потоку = ~ 41 м


33. Если течение в определенный день было в два раза большей скоростью, то время для перехода через реку было бы ____.

а. в два раза больше

г. вдвое меньше

г. больше, но не больше в два раза

г.меньше, но не в два раза меньше исходного времени

e. так же, как это было при скорости тока 1,8 м / с.

Ответ: E

В этой ситуации, когда лодка движется прямо через реку, скорость течения направлена ​​перпендикулярно скорости лодки. Эти две составляющие результирующего движения лодки не зависят друг от друга. Скорость лодки делает единственный вклад в способность лодки пересекать реку.Скорость реки несет лодку только на юг вниз по реке. Таким образом, изменение скорости реки не повлияет на время, необходимое лодке для пересечения реки.

[# 10 | # 11 | # 12 | # 13 | # 14 | # 15 | # 16 | # 17 | # 18 | # 19 | # 20 | # 21 | # 22 | # 23 | # 24 | # 25 | # 26 | # 27 | # 28 | # 29 | # 30 | # 31 | # 32 | # 33 | # 34 | # 35 | # 36 | # 37 | # 38 | # 39 | # 40 | # 41 | # 42 | # 43 | # 44 | # 45]


34. Объект совершает свободное падение. При падении объект ____.

а. скорость увеличивается

г. ускорение увеличивается

г. оба эти

г. ни один из этих

Ответ: A

Когда объект свободно падает, его скорость (а также скорость) изменяется примерно на 10 м / с каждую секунду.Это означает, что ускорение имеет постоянное значение 10 м / с / с. Объект имеет изменяющуюся скорость (или скорость) и постоянное ускорение, если скорость изменяется на одну и ту же величину («постоянную величину») в каждую последующую секунду его движения.

[# 10 | # 11 | # 12 | # 13 | # 14 | # 15 | # 16 | # 17 | # 18 | # 19 | # 20 | # 21 | # 22 | # 23 | # 24 | # 25 | # 26 | # 27 | # 28 | # 29 | # 30 | # 31 | # 32 | # 33 | # 34 | # 35 | # 36 | # 37 | # 38 | # 39 | # 40 | # 41 | # 42 | # 43 | # 44 | # 45]


35.Футбольный мяч подбрасывается в воздух под углом 45 градусов к горизонтали. На самом верху траектории мяча его скорость равна _______.

а. полностью вертикальный

г. полностью горизонтально

г. как вертикальный, так и горизонтальный

г. недостаточно информации, чтобы знать.

Ответ: B

Когда снаряд поднимается к своему пику, его горизонтальная скорость остается постоянной, а вертикальная скорость уменьшается.На пике его вертикальная скорость становится равной 0 м / с. В этот момент скорость полностью горизонтальна; у скорости нет вертикальной составляющей.

[# 10 | # 11 | # 12 | # 13 | # 14 | # 15 | # 16 | # 17 | # 18 | # 19 | # 20 | # 21 | # 22 | # 23 | # 24 | # 25 | # 26 | # 27 | # 28 | # 29 | # 30 | # 31 | # 32 | # 33 | # 34 | # 35 | # 36 | # 37 | # 38 | # 39 | # 40 | # 41 | # 42 | # 43 | # 44 | # 45]


36. Футбольный мяч подброшен в воздух под углом 45 градусов к горизонтали.На самом верху пути мяча его ускорение составляет _______. (Не обращайте внимания на сопротивление воздуха.)

а. полностью вертикальный

г. полностью горизонтально

г. как вертикальный, так и горизонтальный

г. недостаточно информации, чтобы знать.

Ответ: A

Когда снаряд поднимается к своему пику, его горизонтальная скорость остается постоянной, а вертикальная скорость уменьшается.Это означает, что объект ускоряется по вертикали, а не по горизонтали. На пике и по всей траектории наблюдается вертикальное (нисходящее) ускорение. Фактически, снаряд — это объект, на который действует единственная сила тяжести. Эта сила вызывает ускорение в том же направлении, что и сила — вниз.

[# 10 | # 11 | # 12 | # 13 | # 14 | # 15 | # 16 | # 17 | # 18 | # 19 | # 20 | # 21 | # 22 | # 23 | # 24 | # 25 | # 26 | # 27 | # 28 | # 29 | # 30 | # 31 | # 32 | # 33 | # 34 | # 35 | # 36 | # 37 | # 38 | # 39 | # 40 | # 41 | # 42 | # 43 | # 44 | # 45]


37.Футбольный мяч подбрасывается в воздух под углом 45 градусов к горизонтали. В самом верху траектории мяча действующая на него чистая сила равна _______. (Не обращайте внимания на сопротивление воздуха.)

а. полностью вертикальный

г. полностью горизонтально

г. как вертикальный, так и горизонтальный

г.недостаточно информации, чтобы знать.

Ответ: A

Снаряд — это объект, на который действует только сила тяжести. Поскольку никакие другие силы не действуют на объект, результирующая сила будет направлена ​​вниз.

[# 10 | # 11 | # 12 | # 13 | # 14 | # 15 | # 16 | # 17 | # 18 | # 19 | # 20 | # 21 | # 22 | # 23 | # 24 | # 25 | # 26 | # 27 | # 28 | # 29 | # 30 | # 31 | # 32 | # 33 | # 34 | # 35 | # 36 | # 37 | # 38 | # 39 | # 40 | # 41 | # 42 | # 43 | # 44 | # 45]


38.В какой точке его пути горизонтальная составляющая скорости (v x ) снаряда наименьшая?

а. В тот момент, когда он брошен.

г. На полпути к вершине.

г. Наверху.

г. По мере приближения к вершине.

e. То же самое на всем пути.

Ответ: E

Когда снаряд поднимается к своему пику, его горизонтальная скорость остается постоянной, а вертикальная скорость уменьшается. Это означает, что объект ускоряется по вертикали, а не по горизонтали. При постоянной горизонтальной скорости нет точки на траектории, где значение v x меньше, чем в других точках.

[# 10 | # 11 | # 12 | # 13 | # 14 | # 15 | # 16 | # 17 | # 18 | # 19 | # 20 | # 21 | # 22 | # 23 | # 24 | # 25 | # 26 | # 27 | # 28 | # 29 | # 30 | # 31 | # 32 | # 33 | # 34 | # 35 | # 36 | # 37 | # 38 | # 39 | # 40 | # 41 | # 42 | # 43 | # 44 | # 45]


39.В какой точке его пути вертикальная составляющая скорости (v y ) снаряда наименьшая?

а. В тот момент, когда он брошен.

г. На полпути к вершине.

г. Наверху.

г. По мере приближения к вершине.

e. То же самое на всем пути.

Ответ: C

Когда снаряд поднимается к своему пику, его горизонтальная скорость остается постоянной, а вертикальная скорость уменьшается. Во время восходящего участка траектории v y непрерывно уменьшается, пока не станет равным 0 м / с на пике. Таким образом, v y настолько малы, насколько они когда-либо будут, когда они находятся на пике траектории.

[# 10 | # 11 | # 12 | # 13 | # 14 | # 15 | # 16 | # 17 | # 18 | # 19 | # 20 | # 21 | # 22 | # 23 | # 24 | # 25 | # 26 | # 27 | # 28 | # 29 | # 30 | # 31 | # 32 | # 33 | # 34 | # 35 | # 36 | # 37 | # 38 | # 39 | # 40 | # 41 | # 42 | # 43 | # 44 | # 45]


40.Самолет, который летит со скоростью 100 км / ч при боковом ветре урагана со скоростью 100 км / ч, имеет скорость (относительно земли) ____.

а. 0 км / ч

г. 100 км / ч

г. 141 км / ч

г. 200 км / ч

Ответ: C

Когда объект, такой как самолет или лодка, движется в среде, которая движется относительно земли, скорость самолета или лодки (измеренная его спидометром) не будет такой же, как скорость, измеренная человеком. на земле ( путевая скорость ).Скорость относительно земли (т. Е. Результирующая скорость, полученная комбинацией скорости самолета и скорости ветра) может быть определена путем сложения скорости самолета и скорости ветра как векторов. В этом случае два вектора расположены под прямым углом, поэтому результирующую скорость можно определить с помощью теоремы Пифагора.

R 2 = (100 км / час) 2 + (100 км / час) 2

R = SQRT ((100 км / час) 2 + (100 км / час) 2 )

R = 141 км / час

[# 10 | # 11 | # 12 | # 13 | # 14 | # 15 | # 16 | # 17 | # 18 | # 19 | # 20 | # 21 | # 22 | # 23 | # 24 | # 25 | # 26 | # 27 | # 28 | # 29 | # 30 | # 31 | # 32 | # 33 | # 34 | # 35 | # 36 | # 37 | # 38 | # 39 | # 40 | # 41 | # 42 | # 43 | # 44 | # 45]


41.Самолет движется со скоростью 141 км / ч на северо-восток (45 градусов). Какова его составляющая скорость в северном направлении?

а. 41 км / ч

г. 100 км / ч

г. 110 км / ч

г. 141 км / ч

Ответ: B

Чтобы определить компонент вектора в заданном направлении, должны применяться принципы разрешения вектора.Этот вектор скорости направлен под углом 45 градусов и имеет величину 141 км / ч. Вертикальный компонент (север) этого вектора может быть найден с помощью функции синуса.

v y = v * синус (тета)

v y = (141 км / ч) * синус (45 градусов)

v y = 100 км / ч

[# 10 | # 11 | # 12 | # 13 | # 14 | # 15 | # 16 | # 17 | # 18 | # 19 | # 20 | # 21 | # 22 | # 23 | # 24 | # 25 | # 26 | # 27 | # 28 | # 29 | # 30 | # 31 | # 32 | # 33 | # 34 | # 35 | # 36 | # 37 | # 38 | # 39 | # 40 | # 41 | # 42 | # 43 | # 44 | # 45]


42.Скатайте шар для боулинга с края стола. Когда он падает, его горизонтальная составляющая скорости ___.

а. уменьшается

г. остается неизменным

г. увеличивается

Ответ: B

Как только мяч покидает край стола, он становится снарядом. При падении его горизонтальная скорость остается постоянной, а вертикальная — уменьшается.Это означает, что объект ускоряется по вертикали, а не по горизонтали. Фактически, снаряд — это объект, на который действует единственная сила тяжести. Эта сила вызывает ускорение в том же направлении, что и сила — вниз.

[# 10 | # 11 | # 12 | # 13 | # 14 | # 15 | # 16 | # 17 | # 18 | # 19 | # 20 | # 21 | # 22 | # 23 | # 24 | # 25 | # 26 | # 27 | # 28 | # 29 | # 30 | # 31 | # 32 | # 33 | # 34 | # 35 | # 36 | # 37 | # 38 | # 39 | # 40 | # 41 | # 42 | # 43 | # 44 | # 45]


43.Пуля выстреливается горизонтально и падает на землю за 0,5 секунды. Если бы он был запущен с удвоенной скоростью в том же направлении, он бы упал на землю через ____. (Предположим, что нет сопротивления воздуха.)

а. менее 0,5 с.

г. более 0,5 с.

г. 0,5 с.

Ответ: C

Как только пуля выходит из дула, она становится снарядом (при условии отсутствия сопротивления воздуха).При падении его горизонтальная скорость остается постоянной, а вертикальная — уменьшается. Сила тяжести действует на пулю, вызывая ее ускорение вниз. Движение пули в направлении вниз не зависит от движения в горизонтальном направлении. Другими словами, любое изменение в горизонтальном аспекте его движения не повлияет на движение в вертикальном направлении. На время вертикального падения на землю не влияет горизонтальная скорость снаряда.Чтобы упасть на землю с этой высоты, все равно потребуется 0,5 секунды, независимо от горизонтальной скорости.

[# 10 | # 11 | # 12 | # 13 | # 14 | # 15 | # 16 | # 17 | # 18 | # 19 | # 20 | # 21 | # 22 | # 23 | # 24 | # 25 | # 26 | # 27 | # 28 | # 29 | # 30 | # 31 | # 32 | # 33 | # 34 | # 35 | # 36 | # 37 | # 38 | # 39 | # 40 | # 41 | # 42 | # 43 | # 44 | # 45]


44. Снаряд запускается под углом 15 градусов над горизонтом и падает на дальность. При такой же скорости под каким другим углом проецирования будет получено такое же расстояние вниз?

а.30 градусов.

г. 45 градусов.

г. 50 градусов.

г. 75 градусов

e. 90 градусов.

Ответ: D

Для снарядов, выпущенных под углом, угол пуска в 45 градусов обеспечивает наибольшее горизонтальное смещение. Любые два угла запуска, которые отделены от 45 градусов на одинаковую величину (например, 40 градусов и 50 градусов, 30 градусов и 60 градусов и 15 градусов и 75 градусов), будут обеспечивать одинаковое горизонтальное смещение.

[# 10 | # 11 | # 12 | # 13 | # 14 | # 15 | # 16 | # 17 | # 18 | # 19 | # 20 | # 21 | # 22 | # 23 | # 24 | # 25 | # 26 | # 27 | # 28 | # 29 | # 30 | # 31 | # 32 | # 33 | # 34 | # 35 | # 36 | # 37 | # 38 | # 39 | # 40 | # 41 | # 42 | # 43 | # 44 | # 45]


45. Два снаряда выстреливаются с одинаковой скоростью, но под разными углами. Один стреляет под углом 30 градусов, другой — под углом 60 градусов. Снаряд, который первым попадет в землю, будет выпущен (без учета сопротивления воздуха) ____.

а.30 градусов

г. 60 градусов

г. оба попали одновременно

Ответ: A

Для снарядов, выпущенных под углом, угол пуска в 45 градусов обеспечивает наибольшее горизонтальное смещение. Углы пуска более 45 градусов приводят к большим вертикальным составляющим скорости; они дольше остаются в воздухе и поднимаются на большую высоту.Углы пуска менее 45 градусов приводят к небольшим вертикальным составляющим скорости; они не поднимаются так высоко и в конечном итоге за более короткое время падают на землю.

[# 10 | # 11 | # 12 | # 13 | # 14 | # 15 | # 16 | # 17 | # 18 | # 19 | # 20 | # 21 | # 22 | # 23 | # 24 | # 25 | # 26 | # 27 | # 28 | # 29 | # 30 | # 31 | # 32 | # 33 | # 34 | # 35 | # 36 | # 37 | # 38 | # 39 | # 40 | # 41 | # 42 | # 43 | # 44 | # 45]

Перейдите к:

Обзор сессии Главная — Список тем

Векторы и снаряды — Главная || Версия для печати || Вопросы и ссылки

Ответы на вопросы: Все || # 1-9 || # 10-45 || # 46-55 || # 56-72

Вам тоже может понравиться…

Пользователи The Review Session часто ищут учебные ресурсы, которые предоставляют им возможности для практики и обзора, которые включают встроенную обратную связь и инструкции. Если это то, что вы ищете, то вам также может понравиться следующее:
  1. Блокнот калькулятора

    Блокнот калькулятора включает в себя текстовые задачи по физике, сгруппированные по темам. Каждая проблема сопровождается всплывающим ответом и аудиофайлом, в котором подробно объясняется, как подойти к проблеме и решить ее.Это идеальный ресурс для тех, кто хочет улучшить свои навыки решения проблем.

    Визит: Панель калькулятора На главную | Блокнот калькулятора — Векторы и снаряды

  2. Minds On Physics App Series

    Minds On Physics the App («MOP the App») представляет собой серию интерактивных модулей вопросов для учащихся, которые серьезно настроены улучшить свое концептуальное понимание физики. Каждый модуль этой серии посвящен отдельной теме и разбит на подтемы.«Опыт MOP» предоставит учащемуся сложные вопросы, отзывы и помощь по конкретным вопросам в контексте игровой среды. Он доступен для телефонов, планшетов, Chromebook и компьютеров Macintosh. Это идеальный ресурс для тех, кто желает усовершенствовать свои способности к концептуальному мышлению. В первую часть серии входят «Векторы» и «Снаряды».

    Посетите: MOP the App Home || MOP приложение — часть 1

Векторы тяги

Введение

Вектор тяги , \ ({\ bf T} \), это просто вектор силы на поперечном сечении, деленном на площадь этого поперечного сечения.

\ [ {\ bf T} = {{\ bf F} \ over \ text {Area}} \]
Итак, \ ({\ bf T} \) имеет единицы напряжения, такие как МПа , но это абсолютно вектор, а не тензор напряжений. Итак, все обычные правила для векторов применить к нему. Например, скалярные произведения, перекрестные произведения и координаты преобразования могут быть применены.

Расчет вектора тяги

Объект ниже имеет площадь поперечного сечения 400 мм 2 и в настоящее время натягивается силой 4000 Н (красный) в направлении оси x.2 \ right) \ text {4,000} \, {\ bf i} \, \ text {N} = 8.66 \, {\ bf i} \, \ text {МПа} \]
Обратите внимание, что направление вектора тяги всегда совпадает с вектором внутренней силы. Только его величина меняется с углом среза.


Нормальные и касательные напряжения

Нормальные и касательные напряжения — это просто компоненты вектора тяги. которые нормальны и параллельны поверхности участка, как показано на рисунке. Используя \ ({\ bf n} \) в качестве единичного вектора нормали к поверхности, и \ ({\ bf s} \) для параллельного ему единичного вектора означает, что

\ [ \ sigma = {\ bf T} \ cdot {\ bf n} \ qquad \ text {и} \ qquad \ tau = {\ bf T} \ cdot {\ bf s} \]
Очень важно понимать, что \ (\ sigma \) и \ (\ tau \) здесь каждое скалярное значение, а не полные тензоры.Это естественный результат операции скалярного произведения, включающие \ ({\ bf T} \), \ ({\ bf n} \) и \ ({\ bf s} \). (Точечные произведения дают скалярные результаты.)

Значения нормального напряжения и напряжения сдвига здесь являются скалярами, а не тензорами, потому что они являются лишь двумя отдельными компонентами полного тензора напряжений.

Также обратите внимание, что в трехмерном пространстве на самом деле существует бесконечное количество \ ({\ bf s} \) векторы, параллельные поверхности, каждый из которых имеет различную внутреннюю и внешнюю составляющую. страницы, так сказать.\ circ, \; 0) = -4,33 \, \ text {МПа} \]


Тензоры напряжений и векторы тяги

Связь между вектором тяги и напряженным состоянием при балл напрямую зависит от установки суммы силы на объект равны нулю, т. е. устанавливают равновесие.

\ [ \ sigma_ {xx} \, A \, \ cos \ theta + \ tau_ {xy} \, A \, \ sin \ theta = T_x \, ​​A \] \ [ \ tau_ {xy} \, A \, \ cos \ theta + \ sigma_ {yy} \, A \, \ sin \ theta = T_y \, A \]
Область \ (A \) отменяется с обеих сторон, оставляя

\ [ \ sigma_ {xx} \, \ cos \ theta + \ tau_ {xy} \, \ sin \ theta = T_x \] \ [ \ tau_ {xy} \, \ cos \ theta + \ sigma_ {yy} \, \ sin \ theta = T_y \]
, но \ (\ cos \ theta \) и \ (\ sin \ theta \) являются компоненты агрегата перпендикулярны поверхности, \ ({\ bf n} = (\ cos \ theta, \ sin \ theta) \), что \ ({\ bf T} \) действует.

Замена \ (\ cos \ theta \) и \ (\ sin \ theta \) на \ (n_x \) и \ (n_y \) дает

\ [ \ sigma_ {xx} \, n_x + \ tau_ {xy} \, n_y = T_x \] \ [ \ tau_ {xy} \, n_x + \ sigma_ {yy} \, n_y = T_y \]
Оба уравнения можно резюмировать как

\ [ {\ bf T} = \ boldsymbol {\ sigma} \ cdot {\ bf n} \]
или в тензорной записи как

\ [ T_i = \ sigma_ {ij} \, n_j \]
Приведенные выше уравнения являются очень полезными, компактными, матричными и тензорными обозначениями. представления уравнений равновесия.Полные уравнения в 3-D, находятся

\ [ \ sigma_ {xx} \, n_x + \ tau_ {xy} \, n_y + \ tau_ {xz} \, n_z = T_x \] \ [ \ tau_ {yx} \, n_x + \ sigma_ {yy} \, n_y + \ tau_ {yz} \, n_z = T_y \] \ [ \ tau_ {zx} \, n_x + \ tau_ {zy} \, n_y + \ sigma_ {zz} \, n_z = T_z \]
Член тензорной записи \ (\ sigma_ {ij} \, n_j \) приводит к девять отдельных компонентов напряжения. Например, оба \ (\ sigma_ {xz} \) и \ (\ sigma_ {zx} \) присутствуют выше, и оба всегда равны. Фактически это характерно для всех уравнений, включающих напряжение и деформацию.

Вектор тяги от тензора напряжения

Учитывая тензор напряжений (в МПа)

\ [ \ boldsymbol {\ sigma} = \ left [\ matrix { 50 и 10 и 30 \\ 10 и 95 и 20 \\ 30, 20 и 15} \Правильно] \]
Вычислить вектор тяги на поверхности с единичной нормалью. \ ({\ bf n} = (0,400, \, 0,600, \, 0,693) \).

\ [ \ left \ {\ matrix { T_x \ T_y \ T_z} \Правильно\} знак равно \ left [\ matrix { 50 и 10 и 30 \\ 10 и 95 и 20 \\ 30, 20 и 15} \Правильно] \ left \ {\ matrix { 0.400 \ 0,600 \ 0,693} \Правильно\} знак равно \ left \ {\ matrix { 46,79 \ 74,86 \ 34,40} \Правильно\} \]
Итак, \ ({\ bf T} = 46,79 \, {\ bf i} + 74,86 \, {\ bf j} + 34,40 \, {\ bf k} \, \ text {МПа} \).

Если площадь равна 100 мм 2 , то сила на ней будет \ ({\ bf F} = 4 679 \, {\ bf i} + 7 486 \, {\ bf j} + 3 440 \, {\ bf k} \, \ text {N} \).


Преобразование напряжения

В этом разделе представлен аспект преобразования координат тензоров напряжений. это подмножество общего случая, который будет рассмотрен позже.Это делает комбинирование различных уравнений, включающих вектор тяги.

Напомним, что нормальные и касательные напряжения на поверхности связаны к вектору тяги на

\ [ \ sigma = {\ bf T} \ cdot {\ bf n} \ qquad \ text {и} \ qquad \ tau = {\ bf T} \ cdot {\ bf s} \]
Напомним, что нормальные и касательные напряжения здесь — это просто скалярные величины на поверхности, не полный тензор напряжений.

Но мы также видели, что вектор тяги связан с тензором полного напряжения соотношением

\ [ {\ bf T} = \ boldsymbol {\ sigma} \ cdot {\ bf n} \]
Подставляя это уравнение для \ ({\ bf T} \) в приведенные выше, получаем

\ [ \ sigma = {\ bf n} \ cdot \ boldsymbol {\ sigma} \ cdot {\ bf n} \ qquad \ text {и} \ qquad \ tau = {\ bf s} \ cdot \ boldsymbol {\ sigma} \ cdot {\ bf n} \]
В тензорной записи уравнения имеют вид

\ [ \ sigma = \ sigma_ {ij} \, n_i \, n_j \ qquad \ text {и} \ qquad \ tau = \ sigma_ {ij} \, s_i \, n_j \]
Они представляют собой очень полезные взаимосвязи между тензором напряжений в глобальном система координат и компоненты нормального напряжения и напряжения сдвига в любой другой ориентация.

Пример преобразования напряжения

Напомним вышеупомянутый тензор напряжений

\ [ \ boldsymbol {\ sigma} = \ left [\ matrix { 50 и 10 и 30 \\ 10 и 95 и 20 \\ 30, 20 и 15} \Правильно] \]
Мы вычислили вектор тяги на поверхности с единичной нормалью. \ ({\ bf n} = (0,400, \, 0,600, \, 0,693) \). На этот раз посчитайте нормальные и касательные напряжения на этой поверхности.

Нормальное напряжение на поверхности

\ [ \ begin {eqnarray} \сигма & \; знак равно & \ matrix { \ left \ {0.400 \; \; 0,600 \; \; 0,693 \ вправо \} \\ \\ \\ } \ left [\ matrix { 50 и 10 и 30 \\ 10 и 95 и 20 \\ 30, 20 и 15} \Правильно] \ left \ {\ matrix { 0,400 \ 0,600 \ 0,693} \Правильно\} \\ \\ знак равно 87,47 \ text {МПа} \ end {eqnarray} \]
Чтобы вычислить напряжение сдвига, нам сначала нужно конкретное бесконечное количество единичных векторов, параллельных поверхности. Давай выберем \ ({\ bf s} = (-0,832, \, 0,555, \, 0,000) \). Точечный продукт подтвердит, что этот вектор перпендикулярен \ ({\ bf n} \).

\ [ \ begin {eqnarray} \тау & \; знак равно & \ matrix { \ left \ {-0.832 \; \; 0,555 \; \; 0,000 \ вправо \} \\ \\ \\ } \ left [\ matrix { 50 и 10 и 30 \\ 10 и 95 и 20 \\ 30, 20 и 15} \Правильно] \ left \ {\ matrix { 0,400 \ 0,600 \ 0,693} \Правильно\} \\ \\ знак равно 2,62 \ text {МПа} \ end {eqnarray} \]
Таким образом, на этой грани имеется очень небольшой сдвиг в данном направлении s . Но это не значит, что на лице вообще нет среза.Чтобы увидеть это, выберите второе направление параллельно поверхности и перпендикулярно первому s . Получите это путем пересечения единичного вектора нормали с первым касательным вектором.

\ [ \ begin {eqnarray} {\ bf n} \ times {\ bf s} \; знак равно (0,400 \, {\ bf i} + 0,600 \, {\ bf j} + 0,693 \, {\ bf k}) \ times (-0,832 \, {\ bf i} + 0,555 \, {\ bf j} + 0,000 \, {\ bf k}) \\ \\ знак равно -0,385 \, {\ bf i} — 0,576 \, {\ bf j} + 0,721 \, {\ bf k} \\ \ end {eqnarray} \]
Таким образом, сдвиг в направлении, перпендикулярном первому, равен

\ [ \ begin {eqnarray} \тау & \; знак равно & \ matrix { \ left \ {\ text {-} 0.385 \; \; \ text {-} 0,576 \; \; 0,721 \ вправо \} \\ \\ \\ } \ left [\ matrix { 50 и 10 и 30 \\ 10 и 95 и 20 \\ 30, 20 и 15} \Правильно] \ left \ {\ matrix { 0,400 \ 0,600 \ 0,693} \Правильно\} \\ \\ знак равно -36,33 \ text {МПа} \ end {eqnarray} \]
Значит, в этом перпендикулярном направлении имеется хорошее напряжение сдвига. А отрицательное значение указывает на то, что он находится в направлении противоположно направлению s .

Наконечник трансформации

Этот «трюк» с преобразованием можно использовать для вычисления нормалей и сдвига. напряжения на всех шести гранях куба при любой случайной ориентации, а в процесса, выполнить полное преобразование координат тензора напряжений.Т \) так же, как и в случае тензоров деформации.

Но верно и обратное. Тензор напряжений можно заменить на тензор деформации, чтобы получить

\ [ \ epsilon _ {\ text {normal}} \; знак равно {\ bf n} \ cdot \ boldsymbol {\ epsilon} \ cdot {\ bf n} \ qquad \ text {и} \ qquad \ гамма / 2 \; знак равно {\ bf s} \ cdot \ boldsymbol {\ epsilon} \ cdot {\ bf n} \]
Или в тензорной записи как

\ [ \ epsilon _ {\ text {normal}} \; знак равно \ epsilon_ {ij} \, n_i \, n_j \ qquad \ text {и} \ qquad \ гамма / 2 \; знак равно \ epsilon_ {ij} \, s_i \, n_j \]
Это работает, потому что, поскольку и напряжение, и деформация являются тензорами, тогда любые математическая операция, относящаяся к одному, также применима к другому.

\ [ \ тау \; знак равно {\ bf s} \ cdot \ boldsymbol {\ sigma} \ cdot {\ bf n} \; знак равно \ boldsymbol {\ sigma}: ({\ bf s} \ otimes {\ bf n}) \]
(То же самое можно сделать и для вычисления нормального напряжения.)

Это двойное произведение сдвига так часто возникает в пластичности металла, что обозначается одной буквой \ ({\ bf p} \) и называется тензор Шмидта в честь инженера, изучавшего пластичность металлов в начале 1900-х гг.

\ [ {\ bf p} \; знак равно {\ bf s} \ otimes {\ bf n} \; знак равно \ left [\ matrix { s_1 n_1 & s_1 n_2 & s_1 n_3 \\ s_2 n_1 & s_2 n_2 & s_2 n_3 \\ s_3 n_1 & s_3 n_2 & s_3 n_3} \Правильно] \]

\ [ {\ bf F} \; знак равно \ int {\ bf T} \, dA \; знак равно \ int \ boldsymbol {\ sigma} \ cdot {\ bf n} \, dA \]
Обе формы часто встречаются в литературе.

Векторы в двумерных и трехмерных декартовых координатах

Во введении к векторам мы обсуждали векторы без ссылки на какую-либо систему координат. Работая только с геометрическим определением величины и направления векторов, мы смогли определить такие операции, как сложение, вычитание, и умножение на скаляры. Мы также обсудили свойства этих операций.

Часто бывает полезна система координат, потому что проще манипулировать координатами вектора, чем напрямую управлять его величиной и направлением.Когда мы выражаем вектор в системе координат, мы идентифицируем вектор со списком чисел, называемых координатами или компонентами, которые определяют геометрию вектора в терминах системы координат. Здесь мы обсудим стандартные декартовы системы координат на плоскости и в трехмерном пространстве.

Векторы в самолете

Мы предполагаем, что вы знакомы со стандартной декартовой системой координат $ (x, y) $ на плоскости. Каждая точка $ \ vc {p} $ на плоскости отождествляется со своими компонентами $ x $ и $ y $: $ \ vc {p} = (p_1, p_2) $.2} = 5 $.

Приведенный ниже апплет, повторенный из введения вектора, позволяет исследовать взаимосвязь между компонентами вектора и его величиной.

Величина и направление вектора. Синяя стрелка представляет вектор $ \ vc {a} $. Два определяющих свойства вектора, величина и направление, показаны красной полосой и зеленой стрелкой соответственно. Длина красной полосы — это величина $ \ | \ vc {a} \ | $ вектора $ \ vc {a} $. Зеленая стрелка всегда имеет длину один, но ее направление совпадает с направлением вектора $ \ vc {a} $.Единственное исключение — когда $ \ vc {a} $ — нулевой вектор (единственный вектор с нулевой величиной), для которого направление не определено. Вы можете изменить любой конец $ \ vc {a} $, перетащив его мышью. Вы также можете переместить $ \ vc {a} $, перетащив середину вектора; однако изменение положения $ \ vc {a} $ таким образом не меняет вектор, поскольку его величина и направление остаются неизменными.

Подробнее об апплете.

Векторные операции, которые мы определили во введении к векторам, легко выразить через эти координаты.Если $ \ vc {a} = (a_1, a_2) $ и $ \ vc {b} = (b_1, b_2) $, их сумма просто $ \ vc {a} + \ vc {b} = (a_1 + b_1, a_2 + b_2) $, как показано на рисунке ниже. Также легко увидеть, что $ \ vc {b} — \ vc {a} = (b_1-a_1, b_2-a_2) $ и $ \ lambda \ vc {a} = (\ lambda a_1, \ lambda a_2) $ для любого скаляра $ \ lambda $.

Приведенный ниже апплет, также повторяющийся из введения вектора, позволяет исследовать взаимосвязь между геометрическим определением сложения векторов и суммированием компонентов вектора.

Сумма двух векторов. Сумма $ \ vc {a} + \ vc {b} $ вектора $ \ vc {a} $ (синяя стрелка) и вектора $ \ vc {b} $ (красная стрелка) показана зеленой стрелкой. . Поскольку векторы не зависят от их начальной позиции, обе синие стрелки представляют один и тот же вектор $ \ vc {a} $, а обе красные стрелки представляют один и тот же вектор $ \ vc {b} $. Сумму $ \ vc {a} + \ vc {b} $ можно получить, поместив хвост вектора $ \ vc {b} $ в начало вектора $ \ vc {a} $. Точно так же его можно сформировать, поместив хвост вектора $ \ vc {a} $ в начало вектора $ \ vc {b} $.Обе конструкции вместе образуют параллелограмм, причем сумма $ \ vc {a} + \ vc {b} $ является диагональю. (По этой причине коммутативный закон $ \ vc {a} + \ vc {b} = \ vc {b} + \ vc {a} $ иногда называют законом параллелограмма.) Вы можете изменить $ \ vc {a} $ и $ \ vc {b} $, перетаскивая желтые точки.

Подробнее об апплете.

Вы могли заметить, что мы используем одни и те же обозначения для обозначения точки и вектора. Мы не стремимся подчеркивать различие между точкой и вектором.Вы можете представить точку как вектор, хвост которого зафиксирован в начале координат. Вам нужно будет выяснить из контекста, думаем ли мы о векторе как имеющий фиксированный хвост в начале координат.

Другой способ обозначения векторов — это стандартные единичные векторы. обозначается $ \ vc {i} $ и $ \ vc {j} $. Единичный вектор — это вектор, длина которого равна единице. Вектор $ \ vc {i} $ является единичным вектором в направлении положительной оси $ x $. В координатах можно записать $ \ vc {i} = (1,0) $.Аналогично, вектор $ \ vc {j} $ является единичным вектором в направлении положительной оси $ y $: $ \ vc {j} = (0,1) $. Мы можем записать любой двумерный вектор в терминах этих единичных векторов как $ \ vc {a} = (a_1, a_2) = a_1 \ vc {i} + a_2 \ vc {j} $.

Векторы в трехмерном пространстве

В трехмерном пространстве существует стандартная декартова система координат $ (x, y, z) $. Начиная с точки, которую мы называем началом координат, построим три взаимно перпендикулярные оси, которые мы называем осью $ x $, осью $ y $ и осью $ z $.Вот один из способов изобразить эти топоры. Встаньте в углу комнаты и посмотрите вниз, в точку, где стены соприкасаются с полом. Затем пол и стена слева от вас пересекаются по линии, которая является положительной осью $ x $. Пол и стена справа от вас пересекаются по линии, которая является положительной осью $ y $. Стены пересекаются по вертикальной линии, которая является положительной осью $ z $. Эти положительные оси изображены в приведенном ниже апплете и помечены как $ x $, $ y $ и $ z $. Отрицательная часть каждой оси находится на противоположной стороне от начала координат, где оси пересекаются.

Загрузка апплета

Трехмерные декартовы оси координат. Представление трех осей трехмерной декартовой системы координат. Положительная ось $ x $, положительная ось $ y $ и положительная ось $ z $ — это стороны, помеченные символами $ x $, $ y $ и $ z $. Начало координат — это пересечение всех осей. Ветвь каждой оси на противоположной стороне от начала координат (сторона без метки) является отрицательной частью. Вы можете перетащить фигуру мышкой, чтобы повернуть ее.

Подробнее об апплете.

Мы установили относительное расположение положительных осей $ x $, $ y $ и $ z $. чтобы сделать систему координат правой системой координат . Обратите внимание: если вы согнете пальцы правой руки от положительной оси $ x $ к положительной оси $ y $, большой палец правой руки будет указывать в направлении положительной оси $ z $.

Если вы поменяли местами положительную ось $ x $ и положительную ось $ y $, тогда у вас будет левосторонняя система координат .Если вы сделаете это, вы будете жить в математической вселенной, в которой некоторые формулы будут отличаться знаком минус от формулы во вселенной, которую мы здесь используем. Ваша вселенная будет такой же действительной, как и наша, но будет много путаницы. Мы предлагаем вам жить в нашей вселенной, изучая эти страницы.

С помощью этих осей любой точке $ \ vc {p} $ в пространстве можно присвоить три координаты. $ \ vc {p} = (p_1, p_2, p_3) $. Например, учитывая приведенную выше аналогию с углом комнаты, Предположим, вы начали с угла комнаты и прошли четыре метра по оси $ x $, затем повернули налево и прошли три метра в комнату.3 $, чтобы обозначить, что его можно описать тремя действительными координатами. Суммы, разности и скалярные кратные трехмерных векторов выполняются для каждого компонента. Если $ \ vc {a} = (a_1, a_2, a_3) $ и $ \ vc {b} = (b_1, b_2, b_3) $, то $ \ vc {a} + \ vc {b} = (a_1 + b_1, a_2 + b_2, a_3 + b_3) $, $ \ vc {b} — \ vc {a} = (b_1-a_1, b_2-a_2, b_3-a_3) $ и $ \ lambda \ vc {a} = (\ лямбда а_1, \ лямбда а_2, \ лямбда а_3) $.

Загрузка апплета

Вектор в трехмерном пространстве. Представление вектора $ \ vc {a} = (a_1, a_2, a_3) $ в трехмерной декартовой системе координат.Вектор $ \ vc {a} $ изображен в виде зеленой стрелки с хвостом, закрепленным в начале координат. Вы можете перетащить зеленую стрелку с помощью мыши, чтобы изменить вектор. Чтобы показать трехмерную перспективу, розовый треугольник соединяет вектор с его проекцией $ (a_1, a_2,0) $ в плоскости $ xy $ (серая стрелка). Пурпурные векторы показывают проекции $ \ vc {a} $ на каждую ось и представляют координаты $ a_1 $, $ a_2 $ и $ a_3 $. Вы также можете перетащить головы фиолетовых векторов, чтобы изменить только одну из координат вектора.Или перетащите головку серого вектора на плоскости $ xy $, чтобы изменить только координаты $ x $ и $ y $.

Подробнее об апплете.

Так же, как в двух измерениях, мы можем обозначать трехмерные векторы в терминах стандартных единичных векторов: $ \ vc {i} $, $ \ vc {j} $ и $ \ vc {k} $. Эти векторы являются единичными векторами в положительном направлении $ x $, $ y $ и $ z $ соответственно. В терминах координат мы можем записать их как $ \ vc {i} = (1,0,0) $, $ \ vc {j} = (0,1,0) $ и $ \ vc {k} = (0,0,1) $.Мы можем выразить любой трехмерный вектор как сумму скалярных кратных этих единичных векторов в виде $ \ vc {a} = (a_1, a_2, a_3) = a_1 \ vc {i} + a_2 \ vc {j} + a_3 \ vc {k} $.

Загрузка апплета

Стандартные единичные векторы в трех измерениях. Стандартные единичные векторы в трех измерениях: $ \ vc {i} $ (зеленый), $ \ vc {j} $ (синий) и $ \ vc {k} $ (красный) — это векторы длины один, которые указывают параллельно оси $ x $, ось $ y $ и ось $ z $ соответственно. Перемещение их с помощью мыши не меняет векторы, так как они всегда указывают в положительном направлении их соответствующей оси.

Подробнее об апплете.

Какова длина вектора $ \ vc {a} = (a_1, a_2, a_3) $? Мы можем разложить вектор на $ (a_1, a_2, a_3) = (a_1, a_2,0) + (0,0, a_3) $, где два вектора с правой стороны соответствуют два зеленых отрезка линии в приведенном выше апплете. 2} $.n $, где $ n $ — некоторое натуральное число.

Перейти к более высокому измерению легко со списками чисел, хотя, конечно многомерные векторы непросто (невозможно?) визуализировать. Вы можете прочитать больше о многомерных векторах или проверить примеры $ n $ -мерных векторы, которые иллюстрируют, как перейти к размерам больше трех может быть полезно во многих ситуациях.

`

Законы динамики — Энциклопедия окружающей среды

Закон Ньютона, связывающий силу и ускорение, лежит в основе современной физики.Применяемый к каждому графику жидкости, он лежит в основе моделей прогнозирования погоды и климата. Интуитивное понятие силы использовалось с древних времен для понимания статического равновесия. Это позволяет спроектировать своды в архитектуре, использовать рычаг, чтобы описать баланс жидкости под действием тяги Архимеда. Принцип инерции, открытый Галилеем, проложил путь к законам ньютоновской динамики, чей большой успех заключался в объяснении движения планет и спутников, а также приливов и отливов.Это потребовало большого математического опыта, который имеет свои пределы для более сложных систем, таких как атмосфера или океан. Решение уравнений динамики стало возможным только с момента появления компьютерных вычислений. Однако законы сохранения, количества движения, энергии, кинетического момента накладывают глобальные ограничения, позволяющие более непосредственно понять определенные явления.

1. Уравновешивающие силы

Понятие силы выражает механическое воздействие на объект.Силы имеют четко определенное физическое происхождение, например, гравитационная сила (вес ), электрическая сила на заряженной частице , контакт или натяжение сила на кабеле или сила упругости и пружина . Как и многие фундаментальные понятия физики, силу трудно определить сама по себе, но к ней можно подойти с помощью экспериментальных примеров, а также с помощью математических соотношений, которые она имеет с другими величинами.

Рис. 1. Равновесие противоположных сил, а) в пружинном динамометре, б) в воздушном шаре. [Источник: Traité de Physique Elemententaire — DRION et FERNET — 1885] Сила, таким образом, характеризуется своей интенсивностью (или модулем ) и направлением , а также точкой ее приложения и математически представлена ​​как вектор . Сумма сил, действующих на покоящийся объект, должна нейтрализовать друг друга. Сила может быть измерена по удлинению пружины из ее положения покоя (рис. 1).Экспериментально подтверждается, что это удлинение пропорционально силе путем последовательного добавления нескольких одинаковых грузов. После калибровки этот датчик силы пружины можно использовать для измерения различных сил. Для неподвижного в воздухе воздушного шара общий вес уравновешивается тягой Архимеда , равной и противоположной массе вытесненного объема воздуха. Это не что иное, как результат сил атмосферного давления , действующих по всему периметру оболочки: из-за уменьшения давления с высотой давление в нижней части оболочки выше, чем в верхней, что переводится в результирующую силу, направленную вверх. .Этот баланс между давлением и силой тяжести фактически применяется к любому объему жидкости в равновесии, известном как гидростатическое равновесие (см. «Давление, температура, тепло»). Именно благодаря этому балансу воздушные участки или водные участки в бассейне не падают на землю под действием силы тяжести. Когда воздух нагревается, его плотность и, следовательно, масса данного объема уменьшается, в то время как давление остается неизменным, поскольку оно контролируется весом окружающего воздуха.Затем равновесие нарушается, что приводит к вертикальному ускорению воздушного шара. В атмосфере воздушная масса, локально нагретая солнечным излучением, также имеет тенденцию подниматься: это принцип конвекции .

Рис. 2. Перевернутая модель Собора Святого Семейства Гауди. Форма равновесия такова, что на каждом пересечении проводов векторная сумма сил равна нулю. Каждый элемент балки представлен проволокой, а его масса моделируется грузом. Сила натяжения пряжи обязательно совпадает с пряжей.В реальной перевернутой конфигурации соответствующая сила будет тогда силой сжатия, направленной вдоль балки, что гарантирует ее механическую прочность. [Источник: http://olive-art.weebly.com/uploads/2/6/0/5/26053332/6555663_orig.jpg]. В общем, баланс сил должен быть выражен как вектора , что является Основа для расчета структур в архитектуре, см. Рисунок 2. Для каждой материальной точки, например, узла пересечения проводов, векторная сумма сил должна уравновешивать друг друга в состоянии равновесия, как показано на Рисунке 3a.Это позволяет, например, найти интенсивности сил F1 и F2 , зная силу F3 и углы θ1 и θ2 , либо геометрическим построением, либо численно. путем проецирования векторов по вертикальной и горизонтальной осям. Протяженный объект, например твердое тело , описывается в физике как набор из материальных точек , удерживаемых вместе внутренними силами.Эти силы следует отличать от внешних сил, таких как вес или силы контакта с другими объектами. Сумма внутренних сил компенсируется принципом действия и противодействия , так что равновесие требует отмены суммы внешних сил.

Рис. 3. Баланс сил на материальной точке а) и баланс моментов на протяженном объекте — рычаге.

Но состояние равновесия протяженного объекта также требует отмены полного момента сил, чтобы избежать его вращения.Момент силы относительно оси определяется как произведение силы, проецируемой перпендикулярно оси, на расстояние до оси. Классическим примером является рычаг, показанный на рисунке 3b. В состоянии равновесия или квазиравновесия для медленного движения для компенсации моментов требуется, чтобы F1d1 = F2d2 (силы здесь перпендикулярны оси), что позволяет увеличить силу, действующую в обратном отношении расстояния до оси (согласно обычному соглашению мы отмечаем здесь F1 интенсивность силы, а F1 представляет вектор силы).Обычно моменты рассматривают относительно оси рычага, потому что момент силы реакции R земли компенсируется сам собой. Однако тот же результат можно получить, вычислив момент относительно любой математической оси, добавив момент реакции R , который является равным вектором и противоположен сумме двух сил F1 и F2 .

2. Силы и ускорение

Теперь, покидая область статики, ускорение объекта связано с общей силой F , которая действует на него по знаменитому закону ньютоновской динамики , F = мг , где м — масса объекта, г, — его вектор ускорения.Это позволяет определить единицу силы, Ньютон (Н), как силу, создающую ускорение в 1 (м / с) / с над массой 1 кг, что записывается как 1 Н = 1 кг · м · с-2. .

В отсутствие силы объект движется с постоянной скоростью, это принцип инерции , впервые сформулированный Галилеем (1564-1642). В то время этот принцип был не очень интуитивно понятным, потому что в повседневной жизни любое движение имеет тенденцию останавливаться без усилия. Это замедление (отрицательное ускорение) теперь приписывается силам трения силы, которые противоположны скорости.Но трение становится незначительным в межпланетной пустоте, и большой успех Ньютона (1643-1727) заключался в математическом описании движения планет и спутников на основе простого закона всемирной гравитационной силы , убывающей как величина, обратная квадрату расстояния r. .

Применение закона Ньютона потребовало изобретения математической концепции производной , определяющей понятия скорости и ускорения. Отметим скорость v = dz / dt, где dz — небольшое смещение во временном интервале dt.Фактически, мы считаем предел из очень коротким интервалом времени. Точно так же ускорение обозначается как g = dv / dt. Для постоянного ускорения g скорость пропорциональна времени, v = gt , и легко продемонстрировать, что расстояние падения (разница между начальной высотой z0 и высотой z ) будет тогда z0- z = (1/2) gt2 . Таким образом, на Земле под действием силы тяжести г, = 9,8 м / с, объект достигает скорости 9.8 м / с (35 км / ч) за 1 секунду, падает с высоты 4,9 м.

Закон Ньютона применяется в более общем смысле в векторной форме : объект может падать вертикально, как было сказано ранее, сохраняя при этом свою горизонтальную составляющую скорости по инерции. При достаточно высокой горизонтальной скорости необходимо принять во внимание кривизну Земли и получить круговое движение спутника , см. Рис. 4. В этом случае скорость постоянна по модулю, но вектор скорости вращается на такая же угловая скорость [1], что и у спутника.Ускорение тогда перпендикулярно скорости и направлено к центру Земли со значением g = v2 / r , как показано на рисунке 4. Таким образом, для спутника, близкого [2] к Земле, g = 9,8 мс-2, r = 6500 км, что приводит к: скорости v = (gr ) 1/2 = 8 км / с, время обращения (длина 40 000 км) T = 5000 с (1 ч 23 мин).

Рисунок 4. a) Движение запущенного тела с увеличивающейся горизонтальной скоростью до спутника на круговой орбите (8000 м / с) и выброс Земли (11000 м / с) [Источник: иллюстрация теории Ньютона, опубликованная в «Popular Astronomy» ].(б) Закон динамики применительно к круговому движению спутника: за короткий промежуток времени dt спутник поворачивается на угол dq = (v / r) dt, а вектор скорости поворачивается на тот же угол dq = (g / v) dt, из которого выводится g = (v2 / r), что позволяет рассчитать скорость спутника v = 8000 м / с у поверхности Земли (r = 6500 км, g = 9,81 м / с-2). ). Если теперь сила тяжести g уменьшается на 1 / r2, скорость v = (gr ) 1/2 от круговое движение уменьшается на 1 / r1 / 2, а период вращения равен 2πr / v , увеличивается на r3 / 2 .Таким образом Луна удалена от Земли r = 384 000 км, т.е. е. В 60 раз больше радиуса Земли, он должен вращаться за время в 465 раз дольше, чем ближайший спутник, т.е. е. 27 дн. Это согласуется с наблюдениями [3]. Закон роста времени обращения в r3 / 2 был открыт Кеплером (1571-1630) для планет, вращающихся вокруг Солнца. Помимо частного случая круговой орбиты, Ньютон смог продемонстрировать, что общее движение следует за эллипсом (или гиперболой за пределами скорости истечения), и он смог найти три закона, ранее установленные Кеплером, на основе точных измерений положения планет.

Бывает, что сила тяжести сама по себе пропорциональна массе, так что возникающее ускорение не зависит от нее: все тела падают с одинаковым ускорением в одном и том же месте. Это равенство между тяжелой массой и инертной массой было заявлено Галилеем и проиллюстрировано его знаменитыми экспериментами (возможно, только воображаемыми) с падающими объектами с Пизанской башни. Это было сделано снова с большой точностью в вакууме, избегая трения воздуха. В качестве демонстрации для публики был снят фильм, сравнивающий падение молотка и пера на Луне во время полета Аполлона XV [4], и аналогичный эксперимент был снят на Земле в огромной вакуумной камере [5].Именно из-за этого принципа эквивалентности все объекты плавают в невесомости на спутнике, каждый движется по одной и той же орбите вокруг Земли. Эта эквивалентность сейчас проверяется с относительной точностью 10-13 (1/10 триллиона), а точность 10-15 ожидается от недавно запущенного спутника «Микроскоп». Эти сверхточные измерения предназначены для проверки отклонений от принципа эквивалентности, предсказываемого новыми теориями гравитации.

3. Кинетическая и потенциальная энергия

В приведенном выше примере свободного падения можно отметить, что мг (z0-z ) = м (1/2 ) g2t2 = (1/2 ) mv2 .Это соответствует более общему свойству сохранения полной энергии, состоящему из кинетической энергии (1/2 ) mv2 и потенциальной энергии mgz . Когда объект падает, его кинетическая энергия увеличивается, но его потенциальная энергия также уменьшается, так что полная механическая энергия сохраняется (при отсутствии трения). В общем случае кинетическая энергия (1/2 ) mv2 выражается как функция модуля скорости. Сохранение механической энергии становится все более распространенным в случае спутника, но выражение потенциальной энергии должно быть изменено, чтобы учесть уменьшение силы тяжести.Эта потенциальная энергия зависит только от положения объекта, так что он принимает то же значение после вращения, и кинетическая энергия также имеет такое же значение в соответствии с тем фактом, что движение планеты сохраняется бесконечно. Гравитационный потенциал определяется делением этой потенциальной энергии на массу объекта. Этот потенциал характеризует гравитационное поле независимо от объекта, вращающегося вокруг него (до тех пор, пока объект мал и не приводит планету в движение посредством реакции).

Сохранение механической энергии требует особой формы законов силы. Силы трения, в отличие от силы тяжести, уменьшают механическую энергию, так что спутник в конечном итоге входит в контакт с остаточной атмосферой. Однако потерянная механическая энергия преобразуется в тепло, так что общая энергия сохраняется (см. «Энергия»). Тепло по существу соответствует неупорядоченной кинетической энергии молекул газа. Для однородной сферической планеты потенциал изменяется в пределах -1 / r, так что эквипотенциалы представляют собой концентрические сферы.Однако эти сферы слегка деформированы из-за вращения Земли и неоднородностей. Равновесная форма поверхности океана является таким эквипотенциалом (см. «Морская среда»). Действительно, объект, движущийся по эквипотенциалу, сохраняет ту же потенциальную энергию, и, поскольку его полная энергия сохраняется, он не может приобретать (или терять) скорость под действием только силы тяжести. И наоборот, если форма океана отклоняется от эквипотенциальной, поверхностная вода имеет тенденцию течь в области с более низким потенциалом, пока не заполнит их и не достигнет состояния равновесия, при котором поверхность является эквипотенциальной.Поверхность твердой Земли также приближается к эквипотенциальности из-за эрозии и пластичности мантии Земли.

4. Количество движения

Количество движения из элементарной массы (рассматриваемой как точечная) определяется как произведение массы и скорости, определение, которое может быть распространено на любую физическую систему, добавляя (векторно) количества движения каждой из ее элементарные массы. Легко продемонстрировать, что величина движения равна величине движения центра инерции (центра тяжести ) системы, на которую воздействует ее общая масса.Затем закон динамики Ньютона указывает, что производная от количества движения по времени равна сумме сил, действующих на систему.

Согласно фундаментальному принципу физики, количество движения изолированной системы сохраняется. Другими словами, его центр инерции движется поступательно с постоянной скоростью, и только внешние силы могут изменить эту скорость. Другой эквивалентной формулировкой является принцип действия и противодействия , который гласит, что любое тело A, оказывающее силу на тело B, испытывает силу равной интенсивности, но в противоположном направлении, действующую со стороны тела B.Затем закон динамики указывает, что эти внутренние силы не изменяют величину движения глобальной системы A + B. Это обобщает рассмотренное выше условие статического равновесия.

Зная начальные массы m1 и m2 и начальные скорости u1 и u2 каждой массы, вычисляется величина движения до удара m1u1 + m2u2 , которая должна сохраняться после удара , таким образом, обеспечивая ограничение на конечные скорости.Если дополнительно предположить, что амортизатор упругий , т.е. е. что кинетическая энергия (1/2 ) m1u12 + (1/2 ) m2u22 сохраняется, мы можем вывести две конечные скорости. Для двух равных масс мы имеем обмен скоростями (рис. 5а). В случае полностью неупругого удара массы остаются связанными после удара с конечной скоростью, равной средневзвешенному значению начальных скоростей m1u1 + m2u2 / (m1 + m2 ) при сохранении количества движение.Применительно к молекулам газа эти ударные свойства позволяют интерпретировать явление вязкости, которое уравнивает количество движений быстрой и медленной зон внутри жидкости, сохраняя при этом общее количество движений.

Привод ракет или самолетов — еще один классический пример: величина движения, передаваемого транспортному средству, прямо противоположна движению выбрасываемого газа, независимо от задействованных сложных механизмов. Это также относится к силам гравитации: Луна притягивает Землю с силой, равной силе тяжести Земли, действующей на Луну, и противоположной ей.Таким образом, Земля вращается вокруг центра инерции системы Земля-Луна так же, как и молотковая установка, которая должна вращаться, чтобы компенсировать реакцию вращающегося шара (см. «Приливы»). Именно этот центр инерции описывает эллиптическую орбиту вокруг Солнца, а не саму Землю.

Рисунок 5-а. Choc élastique entre deux masses égales [Источник: Саймон Штайнманн (собственная работа) [CC BY-SA 2.5 (http://creativecommons.org/licenses/by-sa/2.5)]] Рис. 5-b.Choc élastique entre deux masses inégales [Источник: Саймон Штайнманн (собственная работа) [CC BY-SA 2.5 (http://creativecommons.org/licenses/by-sa/2.5) через Wikimedia Commons] Рисунок 5-c. Отличное неэластичное блюдо из двух масс [Источник: Par Raul Roque (персонал Travail) [CC BY-SA 2.5 (http://creativecommons.org/licenses/by-sa/2.5)], через Wikimedia Commons]

5. Угловой момент

Угловой момент относительно оси определяется для точечной массы как произведение расстояния до оси на величину ее движения, проецируемого перпендикулярно этой оси.Это определение обобщается на протяженное тело, например твердое тело, путем деления его мыслью на элементарные массы и сложения их угловых моментов. Мы демонстрируем из закона динамики, что производная кинетического момента по времени равна суммарному моменту сил (также называемых « крутящий момент »), действующих на систему. Это обобщает закон статики, который требует, чтобы полный момент сил был равен нулю.

Закон сохранения кинетического момента гласит, что полный момент внутренних сил уравновешивается, и поэтому только момент внешних сил может изменить кинетический момент.Таким образом, в твердом состоянии внутренние силы сцепления не вмешиваются в баланс кинетического момента, так же как они не вмешиваются в величину движения. Это фундаментальный закон физики, отличный от принципа действия и противодействия и дополняющий его.

Другими словами, система не может начать самопроизвольное вращение или потерять свое начальное вращение без действия внешних сил. Однако его скорость вращения может измениться в случае сжатия или растяжения.Действительно, для точечной массы это произведение , скорости u на расстояние r до оси, которая сохраняется, поэтому скорость u увеличивается обратно пропорционально расстоянию r , и его угловая скорость u / r обратно пропорциональна квадрату этого расстояния.

Классический пример — фигурист, а в естественной среде — образование торнадо и циклонов (см. «Торнадо: мощные разрушительные водовороты»).Само вращение Земли является результатом увеличения угловой скорости при аккреции вещества, которое привело к ее образованию. Самый яркий пример — пульсары, чрезвычайно плотные звезды, вращающиеся с периодом от нескольких секунд до нескольких миллисекунд. Эти объекты возникают в результате коллапса массивной звезды, обычно в радиусе от 1 миллиона км до 10 км. Такое сжатие увеличивает угловую скорость вращения в 10 миллиардов раз (часть углового момента выбрасывается вместе с газом, испускаемым взрывом).

Момент импульса на самом деле является вектором, выровненным с осью вращения [6], и поэтому он сохраняется как по направлению, так и по модулю. Это принцип работы гироскопа . Точно так же ось вращения Земли остается выровненной по отношению к звездам, а Северный полюс по-прежнему указывает на область, близкую к Полярной звезде.

Рисунок 6. Прецессия фрезера: вес создает разрез, ориентированный перпендикулярно рисунку и горизонтальный. Таким образом, результирующее изменение вектора кинетического момента перпендикулярно этому вектору, что приводит к прецессионному движению, показанному на рисунке.Аналогичное явление происходит при вращении Земли под действием крутящего момента из-за лунного притяжения (однако, прецессия происходит в противоположном направлении, потому что крутящий момент имеет противоположный знак, чем у вершины). Источник: http://hyperphysics.phy astr.gsu.edu/hbase/mechanics/imgmechs/imgmech/topp.gif]

Это верно только для изолированной системы, а точнее при отсутствии крутящего момента (или момента) от внешние силы. Крутящий момент, перпендикулярный оси вращения, вызывает вращение оси вращения без какого-либо изменения угловой скорости: это явление прецессии , наблюдаемое на маршрутизаторе, см. Рисунок 6 (точно так же, как ускорение, перпендикулярное скорости, производит вращение скорости без изменения ее модуля).Аналогичный эффект происходит с Землей из-за ее уплощенной формы на полюсе: крутящий момент возникает из-за более сильного притяжения Луны на части около бусинки, чем на ее противоположной части. Это приводит к медленному прецессионному движению вращения Земли в течение 26 000 лет (см. Рисунок 6). Таким образом, направление полюса медленно перемещается по небесной сфере на протяжении веков. Это приводит к смещению орбиты Земли относительно положения равноденствий, когда ось вращения Земли ориентирована перпендикулярно направлению Солнца.Вот почему явление называется « прецессия равноденствий ». Связанное с этим изменение солнечного света происходит при изменении климата между ледниковым и умеренным периодами.


Ссылки и примечания

Фотография на обложке. Автор http://creativecommons.org/licenses/by-sa/2.0 (загружен на Flickr как обычный jfpds) [CC BY-SA 2.0 (极 博 双 板 滑雪 俱乐部)] через Wikimedia Commons.

[1] Угловая скорость Ω — это угол, пройденный за единицу времени, обычно выражаемый в радианах / с, так что Ω = v / r.Радиан определяется как угол, пересекающий дугу окружности, равную радиусу, так что полный оборот (окружность 2πr ) представляет 2π радиан, а период вращения равен T = 2π / Ω = 2πr / v .

[2] Высота должна составлять несколько сотен километров, чтобы избежать атмосферного трения, но ускорение свободного падения остается близким к ускорению земной поверхности, а радиус орбиты мало отличается от радиуса Земли.

[3] Это сидеральная революция, i.е. относительно звезд, в то время как время между двумя полными лунами, равное 29,5 дня, является синодическим обращением, i. e . по отношению к Солнцу.

[4] Молот и перо Аполлона 15 — Youtube

[5] Брайан Кокс посещает самую большую в мире вакуумную камеру — Вселенная человека: превью 4 серии — BBC Two — Youtube

[6] Момент импульса более точно определяется относительно исходной точки O. Для точки массой м в точке M это векторное произведение вектора OM на величину движения м и массы в точке М.Для осесимметричного твердого тела, такого как маршрутизатор или Земля, кинетический момент выровнен по оси вращения со значением, пропорциональным угловой скорости и моменту инерции .


Энциклопедия окружающей среды Ассоциации энциклопедий окружающей среды и энергии (www.a3e.fr), по контракту связана с Университетом Гренобль-Альп и ИЯФ Гренобля и спонсируется Французской академией наук.

Для цитирования: SOMMERIA Joël (2021), Законы динамики, Энциклопедия окружающей среды, [онлайн ISSN 2555-0950] URL: https: // www.encyclopedie-environnement.org/en/physics/laws-of-dynamics/.

Статьи в Энциклопедии окружающей среды доступны в соответствии с условиями лицензии Creative Commons BY-NC-SA, которая разрешает воспроизведение при условии: цитирования источника, не коммерческого использования их, использования идентичных исходных условий, воспроизведения в при каждом повторном использовании или распространении эта лицензия Creative Commons BY-NC-SA упоминается.

Home — Инженерный колледж

Инженерный колледж предлагает образовательный опыт мирового класса, который выводит наших студентов за пределы теории и направляет их на реальную, значимую инженерную работу, которая может изменить мир.Мы ценим гостеприимную и благоприятную среду для наших преподавателей, сотрудников и студентов, и мы работаем и учимся в сообществе, которое поощряет разнообразие и инклюзивность. Ознакомьтесь с нашими академическими программами.

Бакалавриат

Инженерный колледж — одно из лучших мест в мире для получения инженерного образования.У нас есть преподаватели мирового класса. У нас есть отличная учебная программа, которая ориентирована на предоставление вам технологических инструментов, ресурсов и знаний, которые помогут вам разработать решения проблем в самых разных областях, от медицины до энергетики и производства — и многих других.

Узнайте больше о наших программах бакалавриата

Выпускные программы

Наши академические программы для выпускников имеют высокие рейтинги, а наши преподаватели пользуются большим уважением как лидеры в своих областях.Здесь вы найдете сообщество, в котором вы преуспеете. Вы найдете преподавателей, сотрудников и однокурсников, которые поддерживают вас и стремятся к вашему успеху. Вы найдете тщательные курсовые работы, которые подготовят вас к достижению ваших целей. Вы окунетесь в атмосферу, способствующую сотрудничеству, и встретитесь с преподавателями с широким спектром исследовательских интересов и связей как в кампусе, так и по всему миру.

Узнайте больше о наших программах магистратуры

Обучение работающих инженеров

Независимо от того, являетесь ли вы новым специалистом или имеете многолетний опыт работы в выбранной вами области, междисциплинарные профессиональные программы UW-Madison могут предложить вам уникальный, высококачественный образовательный опыт, который вы не найдете больше нигде.Наши программы построены таким образом, чтобы помочь работающим специалистам продолжить свой образовательный путь, не отрываясь от своей постоянной работы. После прохождения курсов или получения степени в UW-Madison вы получите больше возможностей, вовлеченности и увлечетесь своей карьерой. И у вас будет уверенность и навыки, чтобы вывести свои проекты или обязанности на новый уровень.

Посмотреть возможности профессионального развития

.

alexxlab

Добавить комментарий

Ваш адрес email не будет опубликован. Обязательные поля помечены *